Hostname: page-component-7bb8b95d7b-w7rtg Total loading time: 0 Render date: 2024-09-27T22:40:31.711Z Has data issue: false hasContentIssue false

Intersecting families without unique shadow

Published online by Cambridge University Press:  02 October 2023

Peter Frankl
Affiliation:
Rényi Institute, Budapest, Hungary
Jian Wang*
Affiliation:
Department of Mathematics, Taiyuan University of Technology, Taiyuan, P. R. China
*
Corresponding author: Jian Wang; Email: wangjian01@tyut.edu.cn
Rights & Permissions [Opens in a new window]

Abstract

Let $\mathcal{F}$ be an intersecting family. A $(k-1)$-set $E$ is called a unique shadow if it is contained in exactly one member of $\mathcal{F}$. Let ${\mathcal{A}}=\{A\in \binom{[n]}{k}\colon |A\cap \{1,2,3\}|\geq 2\}$. In the present paper, we show that for $n\geq 28k$, $\mathcal{A}$ is the unique family attaining the maximum size among all intersecting families without unique shadow. Several other results of a similar flavour are established as well.

MSC classification

Type
Paper
Copyright
© The Author(s), 2023. Published by Cambridge University Press

1. Introduction

Let $n\gt k\gt t$ be positive integers and let $[n]=\{1,2,\ldots,n\}$ be the standard $n$ -element set. For $1\leq i\lt j\leq n$ , let $[i,j]=\{i,i+1,\ldots,j\}$ . Let $\binom{[n]}{k}$ denote the collection of all $k$ -subsets of $[n]$ . Subsets of $\binom{[n]}{k}$ are called $k$ -uniform hypergraphs or $k$ -graphs for short. A $k$ -graph $\mathcal{F}$ is called $t$ -intersecting if $|F\cap F'|\geq t$ for all $F,F'\in{\mathcal{F}}$ . In case of $t=1$ we often use the term intersecting instead of 1-intersecting. Investigating various properties of $t$ -intersecting families is one of the central topics of extremal set theory (cf. the recent book of Gerbner and Patkós [Reference Gerbner and Patkós13]). Let us state the quintessential result of this topic.

Erdős-Ko-Rado Theorem ([Reference Erdős, Ko and Rado3]). Suppose that $n\geq n_0(k,t)$ and ${\mathcal{F}}\subset \binom{[n]}{k}$ is $t$ -intersecting. Then

(1) \begin{align} |{\mathcal{F}}| \leq \binom{n-t}{k-t}. \end{align}

Remark 1. For $t=1$ the exact value $n_0(k,t)=(k-t+1)(t+1)$ was proved in [Reference Erdős, Ko and Rado3]. For $t\geq 15$ it is due to [Reference Frankl5]. Finally Wilson [Reference Wilson21] closed the gap $2\leq t\leq 14$ with a proof valid for all $t$ .

Let us note that the full t-star, $\left \{F\in \binom{[n]}{k}\colon [t]\subset F\right \}$ shows that (1) is best possible. In general, for a set $T\subset [n]$ let ${\mathcal{S}}_T=\left \{S\in \binom{[n]}{k}\colon T\subset S\right \}$ denote the star of T.

For $t=1$ , there is a strong stability for the Erdős-Ko-Rado Theorem.

Theorem 1.1 (Hilton-Milner Theorem [Reference Hilton and Milner14]). Suppose that $n\gt 2k\geq 4$ , ${\mathcal{F}}\subset \binom{[n]}{k}$ is intersecting and $\mathcal{F}$ is not a star, then

(2) \begin{align} |{\mathcal{F}}| \leq \binom{n-1}{k-1}-\binom{n-k-1}{k-1}+1. \end{align}

Let us define the Hilton-Milner Family

\begin{equation*} {\mathcal {H}}(n,k) = \left \{F\in \binom {[n]}{k}\colon 1\in F,\ F\cap [2,k+1]\neq \emptyset \right \}\cup \{[2,k+1]\}, \end{equation*}

showing that (2) is best possible.

Let us recall the notion of immediate shadow, $\partial{\mathcal{F}}$ : For ${\mathcal{F}}\subset \binom{[n]}{k}$ ,

\begin{equation*} \partial {\mathcal {F}} =\left \{G\in \binom {[n]}{k-1}\colon \exists F\in {\mathcal {F}}, G\subset F\right \}. \end{equation*}

If for some $G\in \partial{\mathcal{F}}$ there is only one choice of $F\in{\mathcal{F}}$ satisfying $G\subset F$ then $G$ is called unique or a unique shadow. Note that in the full star ${\mathcal{S}}_{\{x\}}$ for each member $S$ , $S\setminus \{x\}$ is unique. In the Hilton-Milner family ${\mathcal{H}}(n,k)$ , each member $H\in{\mathcal{H}}(n,k)\setminus \{[2,k+1]\}$ contains a unique shadow $H\setminus \{1\}$ . Just for curiosity let us mention that if each member of ${\mathcal{F}}\subset \binom{[n]}{k}$ contains a unique shadow then $|{\mathcal{F}}|\leq \binom{n-1}{k-1}$ .

Let us introduce the central notion of the present paper.

Definition 1.2. For an integer $r\geq 2$ and a family ${\mathcal{F}}\subset \binom{[n]}{k}$ , we say that $\mathcal{F}$ is $r$ -complete if every $G\in \partial{\mathcal{F}}$ is contained in at least $r$ members of $\mathcal{F}$ .

Note that $\mathcal{F}$ is $r$ -complete if and only if the minimum non-zero co-degree of $\mathcal{F}$ is at least $r$ . This notion has been introduced and used by Kostochka et al. [Reference Kostochka, Mubayi and Verstraëte17Reference Kostochka, Mubayi and Verstraëte19] to determine hypergraph Turán numbers for paths, cycles and trees.

Clearly, if ${\mathcal{F}}\subset \binom{[n]}{k}$ is $r$ -complete with $r\geq 2$ , then $\mathcal{F}$ is far from a star. It is natural to ask for the maximum size of an $r$ -complete intersecting family. Let us define the function:

\begin{align*} &f(n,k,r)=\max \left \{|{\mathcal{F}}|\colon{\mathcal{F}}\subset \binom{[n]}{k} \mbox{ is intersecting and } r\mbox{-complete}\right \}. \end{align*}

Let us give some examples. For $1\leq r\lt k$ the complete $k$ -graph $\binom{[k+r]}{k}$ is intersecting and $(r+1)$ -complete. This shows in particular that

(3) \begin{align} f(n,k,k)\geq \binom{2k-1}{k}. \end{align}

Example 1.3. For $n\geq k\geq r\geq 1$ define

\begin{equation*} {\mathcal {L}}(n,k,r) =\left \{F\in \binom {[n]}{k}\colon |F\cap [2r-1]|\geq r\right \}. \end{equation*}

Clearly ${\mathcal{L}}(n,k,r)$ is intersecting, $r$ -complete and

\begin{equation*} |{\mathcal {L}}(n,k,r)|=\sum _{r\leq i\leq 2r-1}\binom {2r-1}{i}\binom {n-2r+1}{k-i}. \end{equation*}

Our main result shows that this example is best possible for $n\geq n_0(k,r)$ .

Theorem 1.4. For $n\geq 28k$ ,

(4) \begin{align} f(n,k,2)=|{\mathcal{L}}(n,k,2)|. \end{align}

Moreover, up to isomorphism ${\mathcal{L}}(n,k,2)$ is the only family attaining equality.

Theorem 1.5. For $k\geq 3$ , $r\geq 3$ and $n\geq n_0(k,r)$ ,

(5) \begin{align} f(n,k,r) = \left \{\begin{array}{l@{\quad}l} |{\mathcal{L}}(n,k,r)|,& 3\leq r\leq k;\\[5pt] 0. &r\geq k+1. \end{array}\right . \end{align}

For a positive integer $\ell$ and an $\ell$ -graph $\mathcal{H}$ , define the clique family

\begin{equation*} {\mathcal {K}}({\mathcal {H}}) =\left \{K\colon |K|=\ell +1,\binom {K}{\ell }\subset {\mathcal {H}}\right \}. \end{equation*}

Define $\nu ({\mathcal{F}})$ , the matching number of $\mathcal{F}$ as the maximum number of pairwise disjoint edges in $\mathcal{F}$ . Note that $\nu ({\mathcal{F}})=1$ iff $\mathcal{F}$ is intersecting. We are going to prove Theorem 1.4 using the following result exhibiting a surprising connection between the matching number and the size of the clique family. Define the Erdős-family

\begin{equation*} {\mathcal {E}}(n,k,s) = \left \{E\in \binom {[n]}{k}\colon E\cap [s]\neq \emptyset \right \}. \end{equation*}

Note that

\begin{equation*} {\mathcal {K}}({\mathcal {E}}(n,k,s))=\left \{K\in \binom {[n]}{k+1}\colon |K\cap [s]|\geq 2\right \}. \end{equation*}

Theorem 1.6. Let ${\mathcal{F}}\subset \binom{[n]}{k}$ be a family with $\nu ({\mathcal{F}})\leq s$ . If $n\geq 5sk+13k$ and $s\geq 3$ , then

\begin{equation*} |{\mathcal {K}}({\mathcal {F}})|\leq |{\mathcal {K}}({\mathcal {E}}(n,k,s))|. \end{equation*}

Moreover, up to isomorphism ${\mathcal{E}}(n,k,s)$ is the only family attaining equality.

Let us define the notion of $r$ -complete edges.

Definition 1.7. For an integer $r\geq 2$ and a family ${\mathcal{F}}\subset \binom{[n]}{k}$ , we say that $F\in{\mathcal{F}}$ is $r$ -complete if every $G\in \binom{F}{k-1}$ is contained in at least $r$ members of $\mathcal{F}$ .

Clearly, $\mathcal{F}$ is $r$ -complete if and only if every $F\in{\mathcal{F}}$ is $r$ -complete. One can also ask for the maximum number of $r$ -complete edges in an intersecting family. For an intersecting family ${\mathcal{F}}\subset \binom{[n]}{k}$ , define ${\mathcal{F}}^*_r({\mathcal{F}})$ as the family of all $r$ -complete edges in $\mathcal{F}$ . Let

\begin{align*} f^*(n,k,r)=\max \left \{|{\mathcal{F}}^*_r({\mathcal{F}})|\colon{\mathcal{F}}\subset \binom{[n]}{k} \mbox{ is intersecting}.\right \} \end{align*}

If $\mathcal{F}$ is $r$ -complete then we have ${\mathcal{F}}^*_r({\mathcal{F}})={\mathcal{F}}$ , implying that $f(n,k,r)\leq f^*(n,k,r)$ . For ${\mathcal{F}}'\subset{\mathcal{F}}$ , we say that ${\mathcal{F}}'$ is relatively $r$ -complete with respect to $\mathcal{F}$ if every $F'\in{\mathcal{F}}'$ is an $r$ -complete edge in $\mathcal{F}$ . Clearly ${\mathcal{F}}^*_r({\mathcal{F}})$ is a relatively $r$ -complete family of the maximum size with respect to $\mathcal{F}$ .

Our next result determines $f^*(n,k,r)$ for all $k\geq 3$ and $r\geq 2$ , asymptotically.

Theorem 1.8. For $k\geq 3$ , $r\geq 2$ and $n\geq n_0(k,r)$ ,

(6) \begin{align} f^*(n,k,r) = \left \{\begin{array}{l@{\quad}l} |{\mathcal{L}}(n,k,r)|,&r=2,3;\\[5pt] \binom{n-3}{k-3}+O(n^{k-r}), &4\leq r\leq k-1;\\[5pt] \binom{n-3}{k-3},& r\geq k\geq 4. \end{array}\right . \end{align}

The next proposition shows that the term $O(n^{k-r})$ in (6) cannot be removed for $k\geq 5$ and $4\leq r\leq k-1$ .

Proposition 1.9. For $k\geq 5$ , $4\leq r\leq k-1$ and $n\geq k+r-1$ ,

\begin{equation*} f^*(n,k,r)\geq \binom {n-3}{k-3}+\binom {n-r-2}{k-r-1}. \end{equation*}

Proof. For $4\leq r\leq k-1$ , let

\begin{equation*} {\mathcal {B}}(r)=\{\{1,2\}\}\cup \left \{\{3,i,j\}\colon i=1\mbox { or } 2,\ 4\leq j\leq r+2\right \}\cup \left \{[r+2]\setminus \{u,3\}\colon u=1,2\right \} \end{equation*}

and let

\begin{equation*} {\mathcal {I}}(n,k,r)=\bigcup _{B\in {\mathcal {B}}(r)}{\mathcal {S}}_B,\ {\mathcal {I}}^*(n,k,r)={\mathcal {S}}_{\{1,2,3\}}\cup {\mathcal {S}}_{[r+2]\setminus \{3\}}. \end{equation*}

It is easy to check that $B(r)$ is intersecting, implying that ${\mathcal{I}}(n,k,r)$ is intersecting. Since ${\mathcal{S}}_{\{1,2,3\}}\subset{\mathcal{S}}_{\{1,2\}}$ and ${\mathcal{S}}_{[r+2]\setminus \{3\}}\subset{\mathcal{S}}_{\{1,2\}}$ , ${\mathcal{I}}^*(n,k,r)\subset{\mathcal{I}}(n,k,r)$ . In the rest of the proof, we show that ${\mathcal{I}}^*(n,k,r)$ is relatively $r$ -complete with respect to ${\mathcal{I}}(n,k,r)$ .

For any $F\in{\mathcal{S}}_{\{1,2,3\}}$ , since $\mathcal{S}_{\{1,2\}}\subset{\mathcal{I}}(n,k,r)$ and $n\geq k+r-1$ , we see that for each $x\in F\setminus \{1,2\}$ , $F\setminus \{x\}$ is covered by at least $r$ members of ${\mathcal{I}}(n,k,r)$ . Moreover, since $\mathcal{S}_{\{3,i,j\}}\subset{\mathcal{I}}(n,k,r)$ for $i\in \{1,2\}$ and $j\in \{4,5,\ldots, r+2\}$ , we infer that $F\setminus \{3-i\}$ is covered by at least $r$ members of ${\mathcal{I}}(n,k,r)$ for $i=1,2$ . Thus ${\mathcal{S}}_{\{1,2,3\}}$ is relatively $r$ -complete with respect to ${\mathcal{I}}(n,k,r)$ .

Let $F\in{\mathcal{S}}_{[r+2]\setminus \{3\}}$ and $G\in \binom{F}{k-1}$ . If $\{1,2\}\subset G$ , then by $\mathcal{S}_{\{1,2\}}\subset{\mathcal{I}}(n,k,r)$ and $n\geq k+r$ we infer that $G$ is covered by at least $r$ members of ${\mathcal{I}}(n,k,r)$ . If $i\notin G$ for $i=1,2$ , since ${\mathcal{S}}_{[r+2]\setminus \{i,3\}}\subset{\mathcal{I}}(n,k,r)$ and $n\geq k+r-1$ , then $G$ is also covered by at least $r$ members of ${\mathcal{I}}(n,k,r)$ . Hence ${\mathcal{S}}_{[r+2]\setminus \{3\}}$ is relatively $r$ -complete with respect to ${\mathcal{I}}(n,k,r)$ . Therefore, ${\mathcal{I}}^*(n,k,r)$ is relatively $r$ -complete with respect to ${\mathcal{I}}(n,k,r)$ and

\begin{equation*} f^*(n,k,r)\geq |{\mathcal {I}}^*(n,k,r)|=\binom {n-3}{k-3}+\binom {n-r-2}{k-r-1}. \end{equation*}

The rest of the paper is organized as follows. We list some results that are needed in Section 2. We prove Theorems 1.4 and 1.6 in Section 3 and prove Theorem 1.5 in Section 4. The proof of Theorem 1.8 splits into two parts. In Section 5, we prove it for $3\leq r\lt k$ . In Section 6, we prove it for $r\geq k$ . Finally, we give some concluding remarks in Section 7.

2. Preliminaries

In this section, we list some notions and results that are needed for the proofs.

For a family ${\mathcal{F}}\subset \binom{[n]}{k}$ define the family of transversals, ${\mathcal{T}}({\mathcal{F}})$ by

\begin{equation*} {\mathcal {T}}({\mathcal {F}}) =\left \{T\subset [n]\colon |T|\leq k,\ T\cap F\neq \emptyset \mbox { for all } F\in {\mathcal {F}}\right \}. \end{equation*}

Note that $\mathcal{F}$ is intersecting iff ${\mathcal{F}}\subset{\mathcal{T}}({\mathcal{F}})$ . Note also that ${\mathcal{T}}({\mathcal{F}})$ is not uniform in general. Set ${\mathcal{T}}^{(k)}({\mathcal{F}}) = \{T\in{\mathcal{T}}({\mathcal{F}})\colon |T|=k\}$ . If ${\mathcal{F}}={\mathcal{T}}^{(k)}({\mathcal{F}})$ then $\mathcal{F}$ is called saturated. It is equivalent to the fact that ${\mathcal{F}}\cup \{H\}$ is no longer intersecting for $H\in \binom{[n]}{k}\setminus{\mathcal{F}}$ . It should be clear that in the definition of $f^*(n,k,r)$ it is sufficient to consider saturated intersecting families $\mathcal{F}$ .

Let us recall a special case of the Katona Intersecting Shadow Theorem [Reference Katona15].

Theorem 2.1 ([Reference Katona15]). Suppose that ${\mathcal{F}}\subset \binom{[n]}{k}$ is intersecting. Then

(7) \begin{align} |\partial{\mathcal{F}}|\geq |{\mathcal{F}}| \mbox{ with equality iff ${\mathcal{F}} =\binom{X}{k}$ for some $(2k-1)$-set }X. \end{align}

We need the following generalization of (7) as well.

Theorem 2.2 ([Reference Frankl8]). Suppose that ${\mathcal{F}}\subset \binom{[n]}{k}$ . Then

(8) \begin{align} |\partial{\mathcal{F}}|\nu ({\mathcal{F}})\geq |{\mathcal{F}}|. \end{align}

We need also a classical result of Bollobás, the so-called Bollobás Set-pair Inequality.

Theorem 2.3 ([Reference Bollobás1]). Let $a,b$ be positive integers, $A_1,\ldots,A_m$ $a$ -element sets, $B_1,\ldots,B_m$ $b$ -element sets such that $A_i\cap B_j=\emptyset$ iff $i=j$ . Then

(9)

There is a very important operation on families of sets which was discovered by Erdős et al. [Reference Erdős, Ko and Rado3]. It is called shifting and it is known not to increase the matching number $\nu ({\mathcal{F}})$ ([Reference Frankl7]) and not to decrease the size of ${\mathcal{K}}({\mathcal{F}})$ (cf. [Reference Liu and Wang20]).

Let us define the shifting partial order $\prec$ . For two $k$ -sets $A$ and $B$ where $A=\{a_1,\ldots,a_k\}$ , $a_1\lt \ldots \lt a_k$ and $B=\{b_1,\ldots,b_k\}$ , $b_1\lt \ldots \lt b_k$ we say that $A$ precedes $B$ and denote it by $A\prec B$ if $a_i\leq b_i$ for all $1\leq i\leq k$ .

A family ${\mathcal{F}}\subset \binom{[n]}{k}$ is called shifted (or initial) if $A\prec B$ and $B\in{\mathcal{F}}$ always imply $A\in{\mathcal{F}}$ . By repeated shifting one can transform an arbitrary $k$ -graph into a shifted $k$ -graph with the same number of edges.

We need the following inequality generalizing the case $t=1$ of the Erdős-Ko-Rado Theorem.

Proposition 2.4 ([Reference Frankl7]). Suppose that ${\mathcal{F}}\subset \binom{[n]}{k}$ then

(10) \begin{align} |{\mathcal{F}}|\leq \nu ({\mathcal{F}})\binom{n-1}{k-1}. \end{align}

Finally we need the following stability theorem concerning the Erdős-Ko-Rado Theorem.

Hilton-Milner-Frankl Theorem ([Reference Frankl6,Reference Hilton and Milner14]). Suppose that ${\mathcal{F}}\subset \binom{[n]}{k}$ is $t$ -intersecting, $\mathcal{F}$ is not a $t$ -star and $n\geq (k-t+1)(t+1)$ . Then

(11) \begin{align} |{\mathcal{F}}|\leq \max \left \{|{\mathcal{A}}(n,k,t)|,\binom{n-t}{k-t}-\binom{n-k-1}{k-t}+t\right \}\lt \binom{n-t-1}{k-t-1}\max \{t+2,k-t+1\}. \end{align}

3. Intersecting families without unique shadow

In this section, we first prove Theorem 1.4 by assuming Theorem 1.6. Then by using the decomposition method of a shifted family introduced in [Reference Frankl9], we give a proof of Theorem 1.6.

Actually, we shall prove the following version of Theorem 1.4, which also gives the $r=2$ case of Theorem 1.8.

Theorem 3.1. For $n\geq 28k$ ,

(12) \begin{align} f(n,k,2)=f^*(n,k,2)=|{\mathcal{L}}(n,k,2)|. \end{align}

Moreover, up to isomorphism ${\mathcal{L}}(n,k,2)$ is the only family attaining equality.

Proof of Theorem 1.4. Recall that ${\mathcal{L}}(n,k,2)$ is 2-complete intersecting and $f(n,k,2)\leq f^*(n,k,2)$ . It follows that $|{\mathcal{L}}(n,k,2)|\leq f(n,k,2)\leq f^*(n,k,2)$ . Thus we are left to show $f^*(n,k,2)\leq |{\mathcal{L}}(n,k,2)|$ .

Let ${\mathcal{F}}\subset \binom{[n]}{k}$ be an intersecting family. Let ${\mathcal{F}}^*$ be the family of 2-complete sets in $\mathcal{F}$ and let ${\mathcal{H}}=\partial{{\mathcal{F}}^*}$ . Note that this guarantees that every member of $\mathcal{H}$ is contained in at least two members of $\mathcal{F}$ .

Claim 1. $\nu ({\mathcal{H}})\leq 3$ .

Proof. Suppose for contradiction that $D_i=F_i\cap G_i$ , $1\leq i\leq 4$ , are pairwise disjoint sets in $\mathcal{H}$ and $F_i,G_i\in{\mathcal{F}}$ . Define $x_i,y_i$ by $F_i\setminus D_i =\{x_i\}$ , $G_i\setminus D_i =\{y_i\}$ . Since $|\{x_i,y_i\}|=2$ , by symmetry we may assume that $(x_1,y_1)\cap D_4= \emptyset$ . This implies $F_1\cap D_4=\emptyset, G_1\cap D_4=\emptyset$ . From $F_4\cap F_1\neq \emptyset$ , $F_4\cap G_1\neq \emptyset$ , $G_4\cap F_1\neq \emptyset$ and $G_4\cap G_1\neq \emptyset$ , we infer $(x_4,y_4)\subset D_1$ . Consequently $F_4\cap D_p=\emptyset$ , $G_4\cap D_p=\emptyset$ for $p=2,3$ . This implies as above $(x_p,y_p)\subset D_4$ . Now $x_2\neq x_3$ or $x_2\neq y_3$ (or both) hold. By symmetry $x_2\neq x_3$ . Then $F_2\cap F_3=\emptyset$ , a contradiction.

By Theorem 1.6 and Claim 1, for $n\geq (5\times 3+13)k=28k$ we have $|{\mathcal{F}}^*|\leq |{\mathcal{K}}({\mathcal{H}})|\leq |{\mathcal{K}}({\mathcal{E}}(n,k,3))|=|{\mathcal{L}}(n,k,2)|$ . The uniqueness follows from Theorem 1.6.

The families ${\mathcal{F}}_0,{\mathcal{F}}_1, \ldots,{\mathcal{F}}_s$ are called overlapping if there is no choice of $F_i \in{\mathcal{F}}_i$ such that $F_0, F_1, \ldots, F_s$ are pairwise disjoint. For the proof of Theorem 1.6 the following lemma is needed. A similar lemma was proved in [Reference Frankl and Wang11], although without characterization of the case of equality.

Lemma 3.2. Let ${\mathcal{F}}_{0}\subset{\mathcal{F}}_{1}\subset \cdots \subset{\mathcal{F}}_{s}\subset \binom{Y}{\ell }$ be overlapping families and let $p_0\geq p_1\geq \ldots \geq p_s$ be positive reals. Let

\begin{equation*} d_{\vec {p}} = \frac {s(p_0+\cdots +p_s)}{p_1+\cdots +p_s}. \end{equation*}

For $|Y|\geq (d_{\vec{p}}+1) \ell$ ,

(13) \begin{align} \sum _{0\leq i\leq s} p_i|{\mathcal{F}}_{i}|\leq (p_1+\cdots +p_s)\binom{|Y|}{\ell }, \end{align}

where the equality holds iff ${\mathcal{F}}_{1}= \cdots ={\mathcal{F}}_{s}=\binom{Y}{\ell }$ , ${\mathcal{F}}_0=\emptyset$ .

Proof. Let ${\mathcal{F}}_0\subset{\mathcal{F}}_1\subset \cdots \subset{\mathcal{F}}_s\subset \binom{Y}{\ell }$ be overlapping families. Let $ t = \lfloor |Y|/\ell \rfloor \geq\lfloor d_{\vec{p}}\rfloor +1\geq s+1$ and choose a random matching $F_1,F_2,\ldots,F_t$ from $\binom{Y}{\ell }$ . Consider the weighted bipartite graph $G$ on partite sets $\{F_1,F_2,\ldots,F_t\}$ and $\{{\mathcal{F}}_0,{\mathcal{F}}_1, \cdots,{\mathcal{F}}_s\}$ where we have an edge $(F_i,{\mathcal{F}}_j)$ iff $F_i\in{\mathcal{F}}_j$ . This edge gets weight $p_j$ .

Since ${\mathcal{F}}_{0}\subset{\mathcal{F}}_{1}\subset \cdots \subset{\mathcal{F}}_{s}$ are overlapping, $G$ has matching number at most $s$ . Applying the König-Hall Theorem we can find $s$ vertices covering all edges of the bipartite graph $G$ . Let $F_1,\ldots,F_q$ be the vertices of the covering set chosen from the random matching and ${\mathcal{F}}_{q+1}, \ldots,{\mathcal{F}}_s$ the remaining $s-q$ chosen from the families.

The total weight of the edges covered by $F_i$ is at most $p_0+\ldots +p_s$ . The total weight of the edges covered by ${\mathcal{F}}_j$ is at most $tp_j$ . Thus, the total weight of the edges in $G$ is at most

(14) \begin{align} q(p_0+\cdots +p_s)+&t(p_{q+1}+\cdots +p_s)\nonumber \\ &= t(p_{1}+\cdots +p_s)- t(p_1+\cdots +p_q)+q(p_0+\cdots +p_s). \end{align}

Note that $p_1\geq \ldots \geq p_s$ implies

\begin{equation*} \frac {i+1}{p_1+\cdots +p_{i+1}}\geq \frac {i}{p_1+\cdots +p_{i}}. \end{equation*}

It follows that

(15) \begin{align} \frac{q}{p_1+\cdots +p_{q}}(p_0+\cdots +p_s)\leq \frac{s}{p_1+\cdots +p_s}(p_0+\cdots +p_s)=d_{\vec{p}}\lt t. \end{align}

By (14) and (15), the total weight of the edges in $G$ is at most $t(p_{1}+\ldots +p_s)$ .

Since the probability

\begin{equation*} Pr(F_i\in {\mathcal {F}}_j) =\frac {|{\mathcal {F}}_j|}{\binom {|Y|}{\ell}}, \end{equation*}

the expected value of the total weight of the edges in $G$ is $\sum _{j=0}^s tp_j \frac{|{\mathcal{F}}_j|}{\binom {|Y|}{\ell}}$ . Thus (13) follows. In case of equality $q=0$ . Then for every $t$ -matching $F_1,F_2,\ldots,F_t$ in $Y$ , ${\mathcal{F}}_0$ has degree 0 and ${\mathcal{F}}_i$ has degree $t$ in $G$ for $i=1,\ldots,s$ . Hence the equality holds iff ${\mathcal{F}}_{1}= \cdots ={\mathcal{F}}_{s}=\binom{Y}{\ell }$ , ${\mathcal{F}}_0=\emptyset$ .

For the proof of Theorem 1.6 we also need the following proposition, which is proved in [Reference Liu and Wang20]. Here we include a short proof for self-containedness.

Proposition 3.3. For ${\mathcal{F}} \subset \binom{[n]}{k}$ and $1\leq i\lt j\leq n$ , $|{\mathcal{K}}(S_{ij}({\mathcal{F}}))|\geq |{\mathcal{K}}({\mathcal{F}})|$ .

Proof. We prove the statement by defining an injective map $\sigma$ from ${\mathcal{K}}({\mathcal{F}})\setminus{\mathcal{K}}(S_{ij}({\mathcal{F}}))$ to ${\mathcal{K}}(S_{ij}({\mathcal{F}}))\setminus{\mathcal{K}}({\mathcal{F}})$ . Let $K\in{\mathcal{K}}({\mathcal{F}})\setminus{\mathcal{K}}(S_{ij}({\mathcal{F}}))$ . Clearly $j\in K$ and $i\notin K$ , and we define $\sigma (K)=K'=(K\setminus \{j\})\cup \{i\}$ . We show that $\sigma$ is well-defined by checking $K'\in{\mathcal{K}}(S_{ij}({\mathcal{F}}))\setminus{\mathcal{K}}({\mathcal{F}})$ . Firstly, suppose that $K'\notin{\mathcal{K}}(S_{ij}({\mathcal{F}}))$ and let $F'\in \binom{K'}{k}$ be an edge not in $S_{ij}({\mathcal{F}})$ . If $i\notin F'$ then $F'=K\setminus \{j\}$ and $S_{ij}(F')=F'$ , implying that $F'\in S_{ij}({\mathcal{F}})$ , a contradiction. If $i\in F'$ , then $F=(F'\setminus \{i\})\cup \{j\}\subset K$ is an edge of $\mathcal{F}$ since $K\in{\mathcal{K}}({\mathcal{F}})$ . Hence after shifting we have $F'\in S_{ij}({\mathcal{F}})$ , a contradiction. This shows $K'\in{\mathcal{K}}(S_{ij}({\mathcal{F}}))$ . Secondly, if $K'\in{\mathcal{K}}({\mathcal{F}})$ then $K\in{\mathcal{K}}({\mathcal{F}})$ implies $K\in{\mathcal{K}}(S_{ij}({\mathcal{F}}))$ , contradicting the assumption that $K\notin{\mathcal{K}}(S_{ij}({\mathcal{F}}))$ . Thus $K'\in{\mathcal{K}}(S_{ij}({\mathcal{F}}))\setminus{\mathcal{K}}({\mathcal{F}})$ and $\sigma$ is indeed a map from ${\mathcal{K}}({\mathcal{F}})\setminus{\mathcal{K}}(S_{ij}({\mathcal{F}}))$ to ${\mathcal{K}}(S_{ij}({\mathcal{F}}))\setminus{\mathcal{K}}({\mathcal{F}})$ . Clearly, $\sigma$ is injective and the proposition follows.

Proof of Theorem 1.6. Since the shifting operator does not increase the matching number and does not decrease the size of ${\mathcal{K}}({\mathcal{F}})$ , we may assume that $\mathcal{F}$ is shifted. Let ${\mathcal{K}}={\mathcal{K}}({\mathcal{F}})$ and ${\mathcal{K}}^*={\mathcal{K}}({\mathcal{E}}(n,k,s))$ . For any $S\subset [s+1]$ and a family ${\mathcal{H}}\subset \binom{[n]}{h}$ , define

\begin{equation*} {\mathcal {H}}(S) \;:\!=\; \left \{H\setminus [s+1]\colon H\in {\mathcal {H}},\ H\cap [s+1]=S\right \}. \end{equation*}

Clearly ${\mathcal{H}}(S)\subset \binom{[s+2,n]}{h-|S|}$ .

For $|S|\geq 3$ , we have ${\mathcal{K}}^*(S)=\binom{[s+2,n]}{k+1-|S|}$ . It follows that

(16) \begin{align} \sum _{S\subset [s+1],|S|\geq 3} |{\mathcal{K}}(S)|\leq \sum _{S\subset [s+1],|S|\geq 3} |{\mathcal{K}}^*(S)|. \end{align}

We are left to compare $|{\mathcal{K}}(S)|$ with $|{\mathcal{K}}^*(S)|$ for all $S\subset [s+1]$ with $|S|\leq 2$ .

Claim 2. ${\mathcal{K}}(\{i\})={\mathcal{F}}(\emptyset )$ for $i=1,2,\ldots,s+1$ and ${\mathcal{K}}(\{i,j\})={\mathcal{F}}(\{j\})$ for $1\leq i\lt j\leq s+1$ .

Proof. For $F\in{\mathcal{K}}(\{i\})$ , $F\cup \{i\}\in{\mathcal{K}}$ implies that $F\in{\mathcal{F}}(\emptyset )$ . Let $F\in{\mathcal{F}}(\emptyset )$ . Since $x\geq s+2\gt i$ each $x\in F$ , by shiftedness $(F\setminus \{x\})\cup \{i\}\in{\mathcal{F}}$ . It follows that $\binom{F\cup \{i\}}{k}\subset{\mathcal{F}}$ and $F\cup \{i\}\in{\mathcal{K}}$ . Thus $F\in{\mathcal{K}}(\{i\})$ . Therefore ${\mathcal{K}}(\{i\})={\mathcal{F}}(\emptyset )$ .

For any $E\in{\mathcal{K}}(\{i,j\})$ we have $\binom{E\cup \{i,j\}}{k}\subset{\mathcal{F}}$ . It follows that $E\cup \{j\}\in{\mathcal{F}}$ . Thus $E\in{\mathcal{F}}(\{j\})$ . Let $E\in{\mathcal{F}}(\{j\})$ . By shiftedness and $i\lt j$ , $E\cup \{i\}\in{\mathcal{F}}$ . Moreover, $E\cup \{i,j\}\setminus \{x\}\in{\mathcal{F}}$ for each $x\in E$ . That is, $\binom{E\cup \{i,j\}}{k}\subset{\mathcal{F}}$ and $E\cup \{i,j\}\in{\mathcal{K}}$ . Thus $E\in{\mathcal{K}}(\{i,j\})$ . Therefore ${\mathcal{K}}(\{i,j\})={\mathcal{F}}(\{j\})$ .

Note that for any $K\in{\mathcal{K}}(\emptyset )$ we have $\binom{K}{k}\subset{\mathcal{F}}(\emptyset )$ . It follows that $\partial{\mathcal{K}}(\emptyset )\subset{\mathcal{F}}(\emptyset )$ . Since $\nu ({\mathcal{K}}(\emptyset )) \leq s$ , by (8) we have

(17) \begin{align} s|{\mathcal{F}}(\emptyset )| \geq s|\partial{\mathcal{K}}(\emptyset )|\geq |{\mathcal{K}}(\emptyset )|. \end{align}

By Claim 2,

\begin{equation*} \sum _{1\leq i\leq s+1}|{\mathcal {K}}(\{i\})|= (s+1)|{\mathcal {F}}(\emptyset )| \end{equation*}

and

\begin{equation*} \sum _{1\leq i\lt j\leq s+1}|{\mathcal {K}}(\{i,j\})|=\sum _{2\leq j\leq s+1}(j-1)|{\mathcal {F}}(\{j\})|. \end{equation*}

It follows that

(18) \begin{align} \sum _{S\in [n],|S|\leq 2} |{\mathcal{K}}(S)| &=|{\mathcal{K}}(\emptyset )|+\sum _{1\leq i\leq s+1}|{\mathcal{K}}(\{i\})|+\sum _{1\leq i\lt j\leq s+1}|{\mathcal{K}}(\{i,j\})|\nonumber \\ &\overset{(17)}{\leq } s|{\mathcal{F}}(\emptyset )|+(s+1)|{\mathcal{F}}(\emptyset )|+\sum _{2\leq j\leq s+1}(j-1)|{\mathcal{F}}(\{j\})|\nonumber \\ &\leq (2s+1)|{\mathcal{F}}(\emptyset )|+\sum _{2\leq j\leq s+1}(j-1)|{\mathcal{F}}(\{j\})|. \end{align}

Again by shiftedness $\partial{\mathcal{F}}(\emptyset )\subset{\mathcal{F}}(\{s+1\})$ , and using (8) we infer

(19) \begin{align} s|{\mathcal{F}}(\{s+1\})| \geq s|\partial{\mathcal{F}}(\emptyset )|\geq |{\mathcal{F}}(\emptyset )|. \end{align}

Substituting (19) into (18), we arrive at

\begin{align*} \sum _{S\in [n],|S|\leq 2} |{\mathcal{K}}(S)| &\leq \sum _{2\leq j\leq s}(j-1)|{\mathcal{F}}(\{j\})|+ s|{\mathcal{F}}(\{s+1\})|+(2s+1)s|{\mathcal{F}}(\{s+1\})|\\[5pt] &= |{\mathcal{F}}(\{2\})|+\sum _{3\leq j\leq s}(j-1)|{\mathcal{F}}(\{j\})|+2s(s+1)|{\mathcal{F}}(\{s+1\})|\\[5pt] &\leq \frac{1}{2}|{\mathcal{F}}(\{1\})|+\frac{1}{2}|{\mathcal{F}}(\{2\})|+\sum _{3\leq j\leq s}(j-1)|{\mathcal{F}}(\{j\})|+2s(s+1)|{\mathcal{F}}(\{s+1\})|. \end{align*}

By shiftedness, ${\mathcal{F}}(\{1\})\supset \cdots \supset{\mathcal{F}}(\{s+1\})$ are overlapping families. Set

\begin{equation*} {\mathcal {F}}_0={\mathcal {F}}(\{s+1\}),\ {\mathcal {F}}_1={\mathcal {F}}(\{s\}),\ \ldots, {\mathcal {F}}_{s-2}={\mathcal {F}}(\{3\}),\ {\mathcal {F}}_{s-1}={\mathcal {F}}(\{2\}),\ {\mathcal {F}}_s={\mathcal {F}}(\{1\}) \end{equation*}

and set

\begin{equation*} p_0=2s(s+1),\ p_1=s-1,\ldots,p_{s-2}=2,\ p_{s-1}=\frac {1}{2},\ p_{s}=\frac {1}{2}. \end{equation*}

Then $p_0\geq p_1\geq \ldots \geq p_s$ and by $s\geq 3$ ,

\begin{equation*} d_{\vec {p}} = \frac {s(p_0+\cdots +p_s)}{(p_1+\cdots +p_s)}=\frac {4s(s+1)+s(s-1)}{s-1}= 5s+ 8 +\frac {8}{s-1}\leq 5s+12. \end{equation*}

By Lemma 3.2, for $n-s-1\geq (5s+13)(k-1)\geq (d_{\vec{p}}+1)(k-1)$ we have

(20) \begin{align} \sum _{S\subset [s+1],|S|\leq 2} |{\mathcal{K}}(S)|&\leq (p_1+p_2+\ldots +p_s)\binom{n-s-1}{k-1}\nonumber \\[5pt]&= \binom{s}{2}\binom{n-s-1}{k-1}\nonumber \\ &= \sum _{S\subset [s+1],|S|\leq 2} |{\mathcal{K}}^*(S)|. \end{align}

Adding (16) and (20), we conclude that

\begin{equation*} |{\mathcal {K}}({\mathcal {F}})|=\sum _{S\subset [s+1]} |{\mathcal {K}}(S)|\leq \sum _{S\subset [s+1]} |{\mathcal {K}}^*(S)| =|{\mathcal {K}}({\mathcal {E}}(n,k,s))|. \end{equation*}

Let $\mathcal{F}$ be a family with $\nu ({\mathcal{F}})\leq s$ and $|{\mathcal{K}}({\mathcal{F}})|= |{\mathcal{K}}({\mathcal{E}}(n,k,s))|$ . If $\mathcal{F}$ is shifted, then by Lemma 3.2 we have ${\mathcal{F}}(\{s+1\})=\emptyset$ . It follows that ${\mathcal{F}}={\mathcal{E}}(n,k,s)$ . Now assume that $\mathcal{F}$ is not shifted. Then it changes to ${\mathcal{E}}(n,k,s)$ by applying shifting repeatedly. Let $\mathcal{G}$ be the last family that is not isomorphic to ${\mathcal{E}}(n,k,s)$ in this process. That is, $\mathcal{G}$ is not isomorphic to ${\mathcal{E}}(n,k,s)$ but $S_{ij}({\mathcal{G}})$ is isomorphic to ${\mathcal{E}}(n,k,s)$ for some $1\leq i\lt j\leq n$ . By symmetry, we may assume that ${\mathcal{G}}\neq{\mathcal{E}}(n,k,s)$ and $S_{s,s+1}({\mathcal{G}})={\mathcal{E}}(n,k,s)$ . Let

\begin{align*} &{\mathcal{G}}(s\overline{(s+1)}) =\left \{E\in \binom{[n]\setminus \{s,s+1\}}{k-1}\colon E\cup \{s\}\in{\mathcal{G}} \right \},\\[5pt] &{\mathcal{G}}(\overline{s}(s+1)) =\left \{E\in \binom{[n]\setminus \{s,s+1\}}{k-1}\colon E\cup \{s+1\}\in{\mathcal{G}} \right \}. \end{align*}

Since $S_{s,s+1}({\mathcal{G}})={\mathcal{E}}(n,k,s)$ , we see ${\mathcal{G}}(s\overline{(s+1)})\cup{\mathcal{G}}(\overline{s}(s+1)) =\binom{[n]\setminus \{s,s+1\}}{k-1}$ and ${\mathcal{G}}(s\overline{(s+1)})\cap{\mathcal{G}}(\overline{s}(s+1)) =\emptyset$ . It follows that for each $E\in \binom{[n]\setminus \{s,s+1\}}{k-1}$ , exactly one of $E\cup \{s\}\in{\mathcal{G}}$ and $E\cup \{s+1\}\in{\mathcal{G}}$ holds. Now consider a graph $G$ on the vertex set $\binom{[n]\setminus \{s,s+1\}}{k-1}$ where $(E_1,E_2)$ forms an edge if and only if $|E_1\cap E_2|=k-2$ . It is easy to see that $G$ is a connected graph. Since $\mathcal{G}$ is not isomorphic to ${\mathcal{E}}(n,k,s)$ , we infer that ${\mathcal{G}}(s\overline{(s+1)})\neq \emptyset$ and ${\mathcal{G}}(\overline{s}(s+1))\neq \emptyset$ . Then there exists an edge $(E_1,E_2)$ in $G$ such that $E_1\cup \{s\}\in{\mathcal{G}}$ and $E_2\cup \{s+1\}\in{\mathcal{G}}$ . Let $F\;:\!=\;E_1\cup E_2\in \binom{[n]\setminus \{s,s+1\}}{k}$ . Then $F\cup \{s+1\}\notin{\mathcal{K}}({\mathcal{G}})$ and $F\cup \{s\}\notin{\mathcal{K}}({\mathcal{G}})$ . But $S_{s,s+1}({\mathcal{G}})={\mathcal{E}}(n,k,s)$ implies $F\cup \{s\} \in{\mathcal{K}}(S_{s,s+1}({\mathcal{G}}))$ . Moreover, for any $K\in{\mathcal{K}}({\mathcal{G}})\setminus{\mathcal{K}}(S_{s,s+1}({\mathcal{G}}))$ , we have $(K\setminus \{s+1\})\cup \{s\}\in{\mathcal{K}}(S_{s,s+1}({\mathcal{G}}))\setminus{\mathcal{K}}({\mathcal{G}})$ by the injective map defined in Proposition 3.3. Hence $|{\mathcal{K}}(S_{s,s+1}({\mathcal{G}}))|\gt |{\mathcal{K}}({\mathcal{G}})|\geq |{\mathcal{K}}({\mathcal{F}})|=|{\mathcal{K}}({\mathcal{E}}(n,k,s))|$ , a contradiction. Thus up to isomorphism ${\mathcal{E}}(n,k,s)$ is the only family attaining equality.

4. The maximum size of an $r$ -complete intersecting family

In this section, we determine $f(n,k,r)$ for all $k,r\geq 3$ and $n\geq n_0(k,r)$ , thereby proving Theorem 1.5.

Proposition 4.1. For $r\geq k+1$ , $f(n,k,r)=0$ . For $n\geq 2k-1$ ,

\begin{equation*} f(n,k,k)=\binom {2k-1}{k}. \end{equation*}

Moreover, the unique family satisfying the condition is $\binom{X}{k}$ with $|X|=2k-1$ .

Proof. Suppose that $\mathcal{F}$ is an intersecting $k$ -graph and each $F\in{\mathcal{F}}$ is $k$ -wise covered. Consider the bipartite graph with partite sets $\mathcal{F}$ , $\partial{\mathcal{F}}$ and an edge between $F$ and $G$ iff $G\subset F$ . It is clear that each $F\in{\mathcal{F}}$ has degree $k$ . On the other hand, the condition implies that each $G\in \partial{\mathcal{F}}$ has degree at least $k$ . Consequently, $|{\mathcal{F}}|\geq |\partial{\mathcal{F}}|$ . In view of (7), we see $|{\mathcal{F}}|= |\partial{\mathcal{F}}|$ and equality holds iff ${\mathcal{F}}=\binom{X}{k}$ with $|X|=2k-1$ . The same argument implies $f(n,k,r)=0$ for $r\geq k+1$ .

We need a notion of basis for an intersecting family inspired by [Reference Frankl6]. For any intersecting family ${\mathcal{F}}\subset \binom{[n]}{k}$ , we define a basis ${\mathcal{B}}({\mathcal{F}})$ which is not necessarily unique by the following process. We start with ${\mathcal{F}}^0={\mathcal{F}}$ . Note that ${\mathcal{F}}^0$ is an antichain. A collection of sets $F_0,\ldots,F_k$ is called a sunflower of size $k+1$ with centre $C$ if $F_i\cap F_j=C$ for all distinct $i,j\in \{0,1,\ldots,k\}$ . Note that in this case $F_0\setminus C,\ldots,F_k\setminus C$ are pairwise disjoint. At the $i$ -th step try and find in the current family ${\mathcal{F}}^i$ a sunflower $F_0,\ldots,F_k$ of size $k+1$ (the size of $F_j$ may be distinct). Let $C_i$ be the centre of the sunflower. Then let ${\mathcal{F}}^{i+1}$ be the family obtained from ${\mathcal{F}}^i$ by deleting all sets containing $C_i$ and adding $C_i$ . Clearly ${\mathcal{F}}^{i+1}$ is also an antichain.

Claim 3. If ${\mathcal{F}}^i$ is intersecting, then ${\mathcal{F}}^{i+1}$ is also intersecting.

Proof. Take an arbitrary set $F$ from ${\mathcal{F}}^{i}$ . Since $|F|\leq k$ , we have $F\cap (F_j\setminus C_i)=\emptyset$ for some $j$ , $0\leq j\leq k$ . Then $F\cap C_i=F\cap F_j\neq \emptyset$ .

Continue this process until no more sunflowers of size $k+1$ can be formed. Let ${\mathcal{B}}({\mathcal{F}})$ be the final family. Clearly, ${\mathcal{B}}({\mathcal{F}})$ is an antichain and for all $F\in{\mathcal{F}}$ there exists $B\in{\mathcal{B}}({\mathcal{F}})$ with $B\subset F$ . By Claim 3, ${\mathcal{B}}({\mathcal{F}})$ is intersecting. In view of the Erdős-Rado sunflower lemma [Reference Erdős and Rado4],

(21) \begin{align} |{\mathcal{B}}^{(\ell )}|\leq \ell !k^\ell, \ \forall 1\leq \ell \leq k. \end{align}

Proof of Theorem 1.5. By proposition 4.1, we may assume $3\leq r\leq k$ . Let $\mathcal{F}$ be an $r$ -complete intersecting family of maximal size and let ${\mathcal{B}}={\mathcal{B}}({\mathcal{F}})$ be its basis. Let $X=\mathop{\cup }\limits _{B\in{\mathcal{B}}} B$ . By (21) we have

\begin{equation*} |X|\leq \sum _{1\leq \ell \leq k} \ell !k^\ell \leq 2k!k^k\leq k^{2k}. \end{equation*}

By the definition of $\mathcal{B}$ , for any $F\in{\mathcal{F}}$ there exists $B\in{\mathcal{B}}$ such that $B\subset F$ . Then for $F,F'\in{\mathcal{F}}$ , there exist $B,B'\in{\mathcal{B}}$ such that $B\subset F$ and $B'\subset F'$ . Since $\mathcal{B}$ is intersecting, $\emptyset \neq B\cap B'\subset F\cap F'\cap X$ . Thus, for all $F,F'\in{\mathcal{F}}$ , $F\cap F'\cap X \neq \emptyset$ .

Let us define $p=\min \left \{|F\cap X|\colon F\in{\mathcal{F}}\right \}$ and choose an arbitrary pair $(F,P_0)$ , $P_0\in \binom{X}{p}$ , $F\cap X=P_0$ . Set $H=F\setminus P_0$ and define

\begin{equation*} {\mathcal {P}}(H) =\left \{P\in \binom {X}{p}\colon H\cup P\in {\mathcal {F}}\right \}. \end{equation*}

Note that $P_0\in{\mathcal{P}}(H)$ .

Claim 4. ${\mathcal{P}}(H)$ is intersecting and $r$ -complete.

Proof. For $P,P'\in{\mathcal{P}}(H)$ fix $B,B'\in{\mathcal{B}}({\mathcal{F}})$ satisfying $B\subset H\cup P, B'\subset H\cup P'$ . Since $H\cap X=\emptyset$ , $B\subset P$ and $B'\subset P'$ . Consequently, $P\cap P'\supset B\cap B'\neq \emptyset$ .

Let us prove the $r$ -completeness of ${\mathcal{P}}(H)$ next. Fix $P\in{\mathcal{P}}(H)$ and $R\in \binom{P}{p-1}$ . Using the $r$ -completeness of $\mathcal{F}$ there are $r$ distinct elements $x_1,x_2,\ldots,x_r$ such that $(H\cup R\cup \{x_i\})\in{\mathcal{F}}$ . The minimal choice of $p$ implies $|(H\cup R\cup \{x_i\})\cap X|\geq p$ , whence $x_i\in X$ , $1\leq i\leq r$ . Thus $R\cup \{x_i\}\in{\mathcal{P}}(H)$ , proving the $r$ -completeness of ${\mathcal{P}}(H)$ .

If $p\lt r$ , by Claim 4 and Proposition 4.1 we have $1\leq |{\mathcal{P}}(H)|\leq f(|X|,p,r)=0$ , a contradiction. Thus $p\geq r$ . Define

\begin{equation*} {\mathcal {F}}_0 =\left \{F\in {\mathcal {F}}\colon |F\cap X|\geq r+1\right \}. \end{equation*}

Then

\begin{equation*} |{\mathcal {F}}_0| \leq \sum _{r+1\leq i\leq k} \binom {|X|}{i}\binom {n-|X|}{k-i}\leq \sum _{r+1\leq i\leq k} \binom {k^{2k}}{i}\binom {n-k^{2k}}{k-i} \lt 2\binom {k^{2k}}{r+1}\binom {n-2r}{k-r-1}. \end{equation*}

If $p\geq r+1$ , then

\begin{equation*} |{\mathcal {F}}|= |{\mathcal {F}}_0| \leq 2\binom {k^{2k}}{r+1}\binom {n-2r}{k-r-1}\leq \binom {2r-1}{r}\binom {n-2r+1}{k-r}\lt |{\mathcal {L}}(n,k,r)|. \end{equation*}

Thus we assume $p=r$ .

If $|{\mathcal{P}}(H)|\leq \binom{2r-1}{r}-1$ holds for all $H\in \binom{[n]\setminus X}{k-r}$ , then

\begin{align*} |{\mathcal{F}}|&\leq \sum _{H\in \binom{[n]\setminus X}{k-r}}|{\mathcal{P}}(H)|+|{\mathcal{F}}_0|\\[5pt] &\leq \left (\binom{2r-1}{r}-1\right )\binom{n-|X|}{k-r}+2\binom{k^{2k}}{r+1}\binom{n-2r}{k-r-1}\\[5pt] &\leq \binom{2r-1}{r}\binom{n-2r+1}{k-r}\ (\mbox{for } n\geq n_0(k,r))\\[5pt] &\lt |{\mathcal{L}}(n,k,r)|. \end{align*}

Assume now that for some $H\in \binom{[n]\setminus X}{k-r}$ , $|{\mathcal{P}}(H)|=\binom{2r-1}{r}$ . By Proposition 4.1 we may assume that ${\mathcal{P}}(H)=\binom{Y}{r}$ , $Y\in \binom{X}{2r-1}$ . We claim that $|F\cap Y|\geq r$ for all $F\in{\mathcal{F}}$ . Indeed the opposite would mean that $F\cap P=\emptyset$ for some $P\in \binom{Y}{r}$ . Then $F\cap (H\cup P)\cap X=F\cap P=\emptyset$ , a contradiction. Consequently ${\mathcal{F}}\subset \{F\in \binom{[n]}{k}\colon |F\cap Y|\geq r\}$ , i.e., $\mathcal{F}$ is contained in an isomorphic copy of ${\mathcal{L}}(n,k,r)$ .

5. Maximizing the number of $r$ -complete sets in an intersecting family

In this section, we prove Theorem 1.8 for $3\leq r\lt k$ and $n\geq n_0(k,r)$ . We need a different notion of basis. For a saturated intersecting family $\mathcal{F}$ , define ${\mathcal{B}}({\mathcal{F}})$ be the family of minimal (for containment) sets in ${\mathcal{T}}({\mathcal{F}})$ . Define $X=\mathop{\cup }\limits _{B\in{\mathcal{B}}} B$ the support of $\mathcal{B}$ . The following properties of ${\mathcal{B}}({\mathcal{F}})$ were proved in [Reference Frankl, Kupavskii and Kiselev10].

Lemma 5.1 ([Reference Frankl, Kupavskii and Kiselev10]). Suppose that ${\mathcal{F}}\subset \binom{[n]}{k}$ is a saturated intersecting family and ${\mathcal{B}}={\mathcal{B}}({\mathcal{F}})$ . Then

  1. (i) $\mathcal{B}$ is an intersecting antichain,

  2. (ii) ${\mathcal{F}}=\{H\in \binom{[n]}{k}\colon \exists B\in{\mathcal{B}}, B\subset H\}$ ,

  3. (iii) for all $F,F'\in{\mathcal{F}}$ ,

    (22) \begin{align} F\cap F'\cap X\neq \emptyset . \end{align}

The following lemma is essentially proved in [Reference Frankl, Kupavskii and Kiselev10]. For self-containedness we include its proof as well.

Lemma 5.2 ([Reference Frankl, Kupavskii and Kiselev10]). Suppose that ${\mathcal{F}}\subset \binom{[n]}{k}$ is a saturated intersecting family. Then $|{\mathcal{B}}({\mathcal{F}})|\leq k^k$ .

Proof. Let ${\mathcal{B}}={\mathcal{B}}({\mathcal{F}})$ . For the proof we use a branching process. During the proof a sequence $S=(x_1,x_2,\ldots,x_\ell )$ is an ordered sequence of distinct elements of $[n]$ and we use $\widehat{S}$ to denote the underlying unordered set $\{x_1,x_2,\ldots,x_\ell \}$ . At the beginning, we assign weight 1 to the empty sequence $S_{\emptyset }$ . At the first stage, we choose $B_1\in{\mathcal{B}}$ with $|B_1|$ minimal. For any vertex $x\in B_1$ , define one sequence $(x)$ and assign the weight $|B_1|^{-1}$ to it.

In each subsequent stage, we pick a sequence $S=(x_1,\ldots,x_p)$ and denote its weight by $w(S)$ . If $\widehat{S}\cap B\neq \emptyset$ holds for all $B\in{\mathcal{B}}$ then we do nothing. Otherwise we pick $B\in{\mathcal{B}}$ satisfying $\widehat{S}\cap B= \emptyset$ and replace $S$ by the $|B|$ sequences $(x_1,\ldots,x_p,y)$ with $y\in B$ and assign weight $\frac{w(S)}{|B|}$ to each of them. Clearly, the total weight is always 1.

We continue until $\widehat{S}\cap B\neq \emptyset$ for all sequences $S$ and all $B\in{\mathcal{B}}$ . Since $[n]$ is finite, each sequence has length at most $n$ and eventually the process stops. Let $\mathcal{S}$ be the collection of sequences that survived in the end of the branching process and let ${\mathcal{S}}^{(\ell )}$ be the collection of sequences in $\mathcal{S}$ with length $\ell$ .

Claim 5. For each $B\in{\mathcal{B}}^{(\ell )}$ , there is some sequence $S\in{\mathcal{S}}^{(\ell )}$ with $\widehat{S}=B$ .

Proof. Let us suppose the contrary and let $S=(x_1,\ldots,x_p)$ be a sequence of maximal length that occurred at some stage of the branching process satisfying $\widehat{S}\subsetneqq B$ . Since $\mathcal{B}$ are intersecting, $B_{1}\cap B\neq \emptyset$ , implying that $p\geq 1$ . Since $\widehat{S}$ is a proper subset of $B$ and $B\in{\mathcal{B}}$ , it follows that $\widehat{S}\notin{\mathcal{T}}({\mathcal{F}})$ . Thereby there exists $F\in{\mathcal{F}}$ with $\widehat{S} \cap F= \emptyset$ . In view of Lemma 5.1 (ii), we can find $B'\in{\mathcal{B}}$ such that $\widehat{S} \cap B'= \emptyset$ . Thus at some point we picked $S$ and some $\tilde{B}\in{\mathcal{B}}$ with $\widehat{S} \cap \tilde{B}= \emptyset$ . Since $\mathcal{B}$ is intersecting, $B\cap \tilde{B}\neq \emptyset$ . Consequently, for each $y\in B\cap \tilde{B}$ the sequence $(x_1,\ldots,x_p,y)$ occurred in the branching process. This contradicts the maximality of $p$ . Hence there is an $S$ at some stage satisfying $\widehat{S}= B$ . Since $\mathcal{B}$ is intersecting, $\widehat{S}\cap B'=B\cap B'\neq \emptyset$ for all $B'\in{\mathcal{B}}$ . Thus $\widehat{S}\in{\mathcal{S}}$ and the claim holds.

By Claim 5, we see that $|{\mathcal{B}}^{(\ell )}|\leq |{\mathcal{S}}^{(\ell )}|$ for all $\ell \geq 1$ . Let $S=(x_1,\ldots,x_\ell )\in{\mathcal{S}}^{(\ell )}$ and let $S_i=(x_1,\ldots,x_i)$ for $i=1,\ldots,\ell$ . At the first stage, $w(S_1)=1/|B_1|$ . Assume that $B_{i}$ is the selected set when replacing $S_{i-1}$ in the branching process for $i=2,\ldots,\ell$ . Then

\begin{equation*} w(S)= \prod _{i=1}^\ell \frac {1}{|B_{i}|} \geq k^{-\ell }. \end{equation*}

It follows that

\begin{equation*} k^{-k}\sum _{1\leq \ell \leq k}|{\mathcal {B}}^{(\ell )}| \leq \sum _{1\leq \ell \leq k}k^{-\ell }|{\mathcal {B}}^{(\ell )}|\leq \sum _{1\leq \ell \leq k}k^{-\ell }|{\mathcal {S}}^{(\ell )}|\leq \sum _{1\leq \ell \leq k}\sum _{S\in {\mathcal {S}}^{(\ell )}}w(S)\leq \sum _{S\in {\mathcal {S}}}w(S)=1. \end{equation*}

Thus $|{\mathcal{B}}|=\sum \limits _{1\leq \ell \leq k}|{\mathcal{B}}^{(\ell )}| \leq k^k$ .

Proposition 5.3. If $\partial{\mathcal{F}}$ is intersecting, then $\mathcal{F}$ is 3-intersecting.

Proof. Suppose that $|F\cap F'|\leq 2$ and $F,F'\in{\mathcal{F}}$ . If $F\cap F'=\{x,x'\}$ then $F\setminus \{x\}, F'\setminus \{x'\}\in \partial{\mathcal{F}}$ and they are disjoint, a contradiction. The case $F\cap F'=\{x\}$ is even easier.

Proposition 5.4. Let ${\mathcal{F}}\subset \binom{[n]}{3}$ be a saturated intersecting family and let ${\mathcal{F}}^*$ be the family of $r$ -complete sets in $\mathcal{F}$ . For $r=3$ , $|{\mathcal{F}}^*|\leq \binom{5}{3}$ with equality holding iff ${\mathcal{F}}=\binom{[5]}{3}$ up to isomorphism. For $r\geq 4$ , $|{\mathcal{F}}^*|\leq 1$ with equality holding iff ${\mathcal{F}}={\mathcal{L}}(n,3,2)$ up to isomorphism.

Proof. Let $r=3$ . Suppose that there exist two edges intersecting in one vertex, say $(x_1,x_2,z), (y_1,y_2,z)\in{\mathcal{F}}^*$ , since $(x_1,x_2)$ is 3-fold covered and $\mathcal{F}$ is intersecting, we have $(x_1,x_2,y_i)\in{\mathcal{F}}$ , $i=1,2$ . Similarly, $(y_1,y_2,x_i)\in{\mathcal{F}}$ , $i=1,2$ . Since $(x_1,x_2,y_2)\in{\mathcal{F}}$ and $(z,y_1)$ is 3-fold covered, $(z,y_1,x_1),(z,y_1,x_2)\in{\mathcal{F}}$ . Similarly, $(z,y_2,x_1),(z,y_2,x_2)\in{\mathcal{F}}$ . Hence $\{x_1,x_2,y_1,y_2,z\}$ spans a complete 3-graph in $\mathcal{F}$ . Since $\mathcal{F}$ is intersecting, we conclude that ${\mathcal{F}}=\binom{[5]}{3}$ up to isomorphism and $|{\mathcal{F}}^*|=10$ . Suppose next that there are two edges intersecting in two vertices say $(x,z_1,z_2),(y,z_1,z_2)\in{\mathcal{F}}^*$ , since $(x,z_1),(y,z_2)$ are 3-fold covered and $\mathcal{F}$ is intersecting, there exists $w$ such that $(x,z_1,y),(y,z_2,x),(x,z_1,w),(y,z_2,w)\in{\mathcal{F}}$ . Arguing with $(x,z_2)$ and $(y,z_1)$ , we infer that $(x,z_2,y),(y,z_1,x),(x,z_2,w),(y,z_1,w)\in{\mathcal{F}}$ . Hence $\{x,y,z_1,z_2,w\}$ spans a complete 3-graph in $\mathcal{F}$ . Since $\mathcal{F}$ is intersecting, we conclude that ${\mathcal{F}}=\binom{[5]}{3}$ up to isomorphism and $|{\mathcal{F}}^*|=10$ . If ${\mathcal{F}}^*$ is 3-intersecting, then $|{\mathcal{F}}^*|\leq 1$ holds trivially.

For $r\geq 4$ , we claim that each member in $\partial{{\mathcal{F}}^*}$ is a transversal of $\mathcal{F}$ . Otherwise, let $G\in \partial{{\mathcal{F}}^*}$ be a $2$ -set that is not a transversal. Then there exists $F\subset{\mathcal{F}}$ such that $F\cap G=\emptyset$ . Since $G\in \partial{\mathcal{F}}^*$ and $r\geq 4\gt |F|$ , there exists $x$ such that $G\cup \{x\}\in{\mathcal{F}}$ and $F\cap (G\cup \{x\})=\emptyset$ , a contradiction.

Thus $\partial{{\mathcal{F}}^*}\subset{\mathcal{T}}({\mathcal{F}})$ . By Lemma 5.1 (i) $\partial{{\mathcal{F}}^*}$ is intersecting. In view of Proposition 5.3, ${\mathcal{F}}^*$ is 3-intersecting. For $n\geq 6$ , by Proposition 5.3 and (1) we have $|{\mathcal{F}}^*|\leq \binom{n-3}{3-3}=1$ . In the case of equality, by symmetry we may assume that ${\mathcal{F}}^*=[3]$ .

Then we claim $|F\cap [3]|\geq 2$ for all $F\in{\mathcal{F}}$ . Indeed, otherwise $|F\cap [3]|=1$ for some $F\in{\mathcal{F}}$ , without loss of generality assume $F\cap [3]=\{1\}$ , then we can find an $F'\in{\mathcal{F}}$ disjoint to $F$ since $(2,3)$ is $r$ -fold covered with $r\geq 4\gt |F|$ , a contradiction. Thus $|F\cap [3]|\geq 2$ for all $F\in{\mathcal{F}}$ . Using that $\mathcal{F}$ is saturated, we conclude that ${\mathcal{F}}={\mathcal{L}}(n,3,2)$ up to isomorphism. For $n\leq 5$ , clearly ${\mathcal{F}}\subset \binom{[5]}{3}$ . Since no 2-set is contained in 4 or more 3-sets in $\binom{[5]}{3}$ , we have $|{\mathcal{F}}^*|=0$ .

The following proposition proves Theorem 1.8 for $3\leq r\leq k-1$ and $n\geq n_0(k,r)$ .

Proposition 5.5. $f^*(n,k,r) =\binom{n-3}{k-3}+O(n^{k-r})$ for $4\leq r\leq k-1$ and $n\geq n_0(k,r)$ ; $f^*(n,k,3) =|{\mathcal{L}}(n,k,3)|$ for $n\geq n_0(k)$ .

Proof. Let ${\mathcal{F}}\subset \binom{[n]}{k}$ be a saturated intersecting family and let ${\mathcal{F}}^*$ be the family of $r$ -complete sets in $\mathcal{F}$ . Let ${\mathcal{B}}={\mathcal{B}}({\mathcal{F}})$ and $X=\mathop{\cup }\limits _{B\in{\mathcal{B}}} B$ . By Lemma 5.2, we have $|X|\leq k\cdot k^k=k^{k+1}$ . Let

\begin{equation*} p=\min \left \{|F\cap X|\colon F\in {\mathcal {F}}^*\right \}. \end{equation*}

If $p\geq 4$ , then for $n\geq n_0(k,r)$ we have

\begin{equation*} |{\mathcal {F}}^*|\leq \sum _{4\leq i\leq k} \binom {|X|}{i}\binom {n-|X|}{k-i}\leq \sum _{4\leq i\leq k} \binom {k^{k+1}}{i}\binom {n-k^{k+1}}{k-i} \leq 2\binom {k^{k+1}}{4}\binom {n-6}{k-4}\lt \binom {n-3}{k-3} \end{equation*}

and we are done. If $p=1$ , then there exists $F^*\in{\mathcal{F}}^*$ such that $F^*\cap X=\{x\}$ . It follows that $\{x\}\in{\mathcal{B}}$ . By saturatedness we have ${\mathcal{F}}={\mathcal{S}}_x$ and $|{\mathcal{F}}^*|=0$ . If $p=2$ then for some $F^*\in{\mathcal{F}}^*$ , $F^*\cap X=\{x,y\}\in{\mathcal{B}}$ . Using $r$ -completeness we find $F_x\in{\mathcal{F}}$ , $F_x\supset F^*\setminus \{y\}$ and $F_y\in{\mathcal{F}}$ , $F_y\supset F^*\setminus \{x\}$ and $F_x\cap F_y\cap X=\emptyset$ , contradicting (22). Thus we may assume $p=3$ .

Define the 3-graph

\begin{equation*} {\mathcal {T}}^* =\left \{T\in \binom {X}{3}\colon \exists F^*\in {\mathcal {F}}^*, F^*\cap X=T\right \},\ {\mathcal {T}} =\left \{T\in \binom {X}{3}\colon \exists F\in {\mathcal {F}}, F\cap X=T\right \}. \end{equation*}

Note that $p=3$ implies ${\mathcal{T}}\neq \emptyset$ . By (22) we infer that $\mathcal{T}$ is intersecting. We distinguish two cases.

Case 1. $r=3$ .

If there exist $(x_1,y_1,z), (x_2,y_2,z)\in{\mathcal{T}}^*$ , let $H_i\in \binom{[n]\setminus X}{k-3}$ such that $H_i\cup \{x_i,y_i,z\}\in{\mathcal{F}}^*$ , $i=1,2$ . By 3-completeness and (22), we have

\begin{equation*} (x_1,y_1,x_2),(x_1,y_1,y_2),(x_2,y_2,x_1),(x_2,y_2,y_1)\in {\mathcal {T}}. \end{equation*}

Then there exists $H_3\in \binom{[n]\setminus X}{k-3}$ such that $H_3\cup \{x_1,y_1,x_2\}\in{\mathcal{F}}$ . Since $H_2\cup \{y_2,z\}$ is covered by at least 3 members of $\mathcal{F}$ , by (22) we infer $H_2\cup (x_1,y_2,z),H_2\cup (y_1,y_2,z)\in{\mathcal{F}}$ . Similarly, we have $H_2\cup (x_2,y_1,z),H_2\cup (x_1,x_2,z)\in{\mathcal{F}}$ . Hence $\{x_1,x_2,y_1,y_2,z\}$ spans a complete 3-graph in $\mathcal{T}$ . We claim that $|F\cap \{x_1,x_2,y_1,y_2,z\}|\geq 3$ for all $F\in{\mathcal{F}}$ . Indeed, otherwise suppose that there is $F\in{\mathcal{F}}$ with $|F\cap \{x_1,x_2,y_1,y_2,z\}|\leq 2$ . Without loss of generality assume that $F\cap \{y_1,y_2,z\}=\emptyset$ . Since $\{y_1,y_2,z\}\in{\mathcal{T}}$ , there exists $H\in \binom{[n]\setminus X}{k-3}$ such that $H\cup \{y_1,y_2,z\}=:F'\in{\mathcal{F}}$ . But then $F\cap F'\cap X=\emptyset$ , contradicting (22). By saturatedness, we conclude that ${\mathcal{F}}={\mathcal{L}}(n,k,3)$ up to isomorphism and $|{\mathcal{F}}^*|= |{\mathcal{L}}(n,k,3)|$ .

If there exist $(x_1,y,z), (x_2,y,z)\in{\mathcal{T}}^*$ , let $H_i\in \binom{[n]\setminus X}{k-3}$ such that $H_i\cup \{x_i,y,z\}\in{\mathcal{F}}^*$ , $i=1,2$ . Since $H_1\cup \{x_1,y\},H_2\cup \{x_2,z\}$ are 3-fold covered, by (22) there exists $w\in X$ such that $H_1\cup \{x_1,y,x_2\},H_1\cup \{x_1,y,w\},H_2\cup \{x_2,z,x_1\},H_2\cup \{x_2,z,w\}\in{\mathcal{F}}$ . Similarly, $H_1\cup \{x_1,z,x_2\},H_1\cup \{x_1,z,w\},H_2\cup \{x_2,y,x_1\},H_2\cup \{x_2,y,w\}\in{\mathcal{F}}$ . Then $\{x_1,x_2,y,z,w\}$ spans a complete 3-graph in $\mathcal{T}$ . By the same argument and saturatedness, we conclude that ${\mathcal{F}}={\mathcal{L}}(n,k,3)$ up to isomorphism and $|{\mathcal{F}}^*|= |{\mathcal{L}}(n,k,3)|$ .

Now we may assume that ${\mathcal{T}}^*$ is 3-intersecting. Since ${\mathcal{T}}^*$ is a 3-graph, we trivially have $|{\mathcal{T}}^*|\leq 1$ . Then for $n\geq n_0(k)$ we obtain that

\begin{equation*} |{\mathcal {F}}^*| \leq \binom {n-|X|}{k-3}+\sum _{4\leq i\leq k} \binom {|X|}{i}\binom {n-|X|}{k-i} \leq 10\binom {n-5}{k-3} \leq |{\mathcal {L}}(n,k,3)|. \end{equation*}

Case 2. $r\geq 4$ .

Claim 6. For all $F\in{\mathcal{F}}^*$ and $T\in{\mathcal{T}}^*$ ,

(23) \begin{align} |F\cap T|\geq 2. \end{align}

Proof. Suppose the contrary. By symmetry let $T=\{1,2,3\}$ , $F\cap T=\{3\}$ ( $F\cap T\neq \emptyset$ by (22)). By $r$ -completeness there are distinct elements $y_1,\ldots,y_r$ such that $(F\setminus \{3\})\cup \{y_i\}\in{\mathcal{F}}$ . Since $r\geq 4$ , without loss of generality, assume $y_r\notin \{1,2,3\}$ . Then $((F\setminus \{3\})\cup \{y_r\})\cap T=\emptyset$ contradicting (22).

Claim 7. $|{\mathcal{T}}^*|=1$ .

Proof. Otherwise using Claim 6, without loss of generality, $\{1,2,3\},\{1,2,4\}\in{\mathcal{T}}^*$ . Let $H_i\in \binom{[n]\setminus X}{k-3}$ such that $H_i\cup \{1,2,i\}\in{\mathcal{F}}^*$ , $i=3,4$ . Let $x_1,\ldots,x_r$ be such that $H_3\cup \{1,3,x_j\}\in{\mathcal{F}}$ , $j=1,\ldots,r$ . Let $y_1,\ldots,y_r$ be such that $H_4\cup \{2,4,y_j\}\in{\mathcal{F}}$ , $j=1,\ldots,r$ . By $r\geq 4$ , without loss of generality assume $x_1\notin \{2,4\}$ and $y_1\notin \{1,3,x_1\}$ . Then

\begin{equation*} (H_3\cup \{1,3,x_1\})\cap (H_4\cup \{2,4,y_1\})\cap X=\emptyset, \end{equation*}

contradicting (22).

By Claim 7, we may assume that ${\mathcal{T}}^*=\{(1,2,3)\}$ . Define

\begin{equation*} {\mathcal {F}}_i^* =\{F\in {\mathcal {F}}^*\colon F\cap [3]=[3]\setminus \{i\}\},\ i=1,2,3. \end{equation*}

Claim 8. $F\in{\mathcal{F}}_i^*$ implies $|F\cap X|\geq r$ , $i=1,2,3$ .

Proof. By symmetry assume $i=1$ and set $S=F\cap X$ . Suppose indirectly $|S|\lt r$ . Let $\tilde{F}\in{\mathcal{F}}^*$ with $\tilde{F}\cap X=[3]$ . By $r$ -completeness there are $x_1,x_2,\ldots,x_r$ distinct elements with $(\tilde{F}\setminus \{3\})\cup \{x_j\}\in{\mathcal{F}}$ , $1\leq j\leq r$ . Also $F\setminus \{2\}$ is contained in $r\gt 3$ members of $\mathcal{F}$ . Let $\hat{F}$ be one of them with $\hat{F}\cap [3] = \{3\}$ and let $\hat{S}= \hat{F}\cap X$ . Clearly, $|\hat{S}|\leq |S|\lt r$ . Consequently we can choose $x_j\notin \hat{S}$ . Then

\begin{equation*} \left (\left (\tilde {F}\setminus \{3\}\right )\cup \{x_j\}\right )\cap \hat {F}\cap X=\emptyset, \end{equation*}

contradicting (22).

By Claims 6, 7, 8, we have

\begin{align*} |{\mathcal{F}}^*|&\leq |{\mathcal{T}}^*|\binom{n-|X|}{k-3}+\sum _{1\leq i\leq 3}|{\mathcal{F}}_i^*|\\[5pt] &\leq \binom{n-|X|}{k-3}+\sum _{r\leq i\leq k} \binom{3}{2}\binom{|X|}{i-2}\binom{n-|X|}{k-i}\\[5pt] &=\binom{n-3}{k-3}+O(n^{k-r}) \end{align*}

and the proposition is proven.

6. Maximizing the number of $k$ -complete sets in an intersecting family

In this section, we prove Theorem 1.8 for $r\geq k$ . By using Bollobás Set-pairs Inequality (Theorem 2.3) and the Hilton-Milner-Frankl Theorem, we determine $f^*(n,k,k)$ for $k\geq 5$ and $n\geq n_0(k)$ . The cases $r\geq k+1$ and $r=k=4$ of Theorem 1.8 will be proved separately.

First we show that if an intersecting family contains a relatively $k$ -complete sunflower of given shape, then Theorem 1.8 holds.

Lemma 6.1. Let ${\mathcal{F}}\subset \binom{[n]}{k}$ be an intersecting family and let ${\mathcal{F}}^*$ be the family of $k$ -complete sets in $\mathcal{F}$ . If ${\mathcal{F}}^*$ contains a sunflower with $k+1$ petals and centre $C$ of size $3$ and $k\geq 4$ , then $C\subset F$ for all $F\in{\mathcal{F}}^*$ . In particular,

\begin{equation*} |{\mathcal {F}}^*|\leq \binom {n-3}{k-3}. \end{equation*}

Proof. Suppose that $F_1,F_2,\ldots,F_{k+1}$ is a sunflower in ${\mathcal{F}}^*$ with centre $[3]$ and let $G_i=F_i\setminus [3]$ , $i=1,\ldots,k+1$ .

If there exists $F\in{\mathcal{F}}^*$ with $|F\cap [3]|\leq 1$ , pick $G\in \binom{F}{k-1}$ with $G\cap [3]=\emptyset$ . Then $G\cap F_i\neq \emptyset$ can hold for at most $k-1$ values of $i$ . Pick $F_p,F_r$ disjoint to $G$ . Now $k$ -completeness and $k\geq 4$ imply that we can choose $z\notin [3]$ , $G\cup \{z\}\in{\mathcal{F}}$ . Then either $F_p$ or $F_r$ is disjoint to $G\cup \{z\}$ , a contradiction.

If there exists $F\in{\mathcal{F}}^*$ with $|F\cap [3]|= 2$ , without loss of generality, assume that $F\cap [3]= \{1,2\}$ and let $G=F\setminus \{1,2\}$ . Pick $F_p,F_q,F_r$ disjoint to $G$ . Since $k\geq 4$ , we can choose $z,w\notin [3]$ such that $G\cup \{1,z\},G\cup \{1,w\}\in{\mathcal{F}}$ . Then one of $F_p, F_q, F_r$ , without loss of generality say $F_p$ , is disjoint to both $G\cup \{z\}$ and $G\cup \{w\}$ . Since $F_p\setminus \{1\}$ is covered by at least $k$ members of $\mathcal{F}$ , we can find $u\notin G\cup \{1\}$ such that $(F_p\setminus \{1\})\cup \{u\}\in{\mathcal{F}}$ . Then either $z\neq u$ or $w\neq u$ holds. Without loss of generality, assume that $z\neq u$ , then $(F_p\setminus \{1\})\cup \{u\}$ and $G\cup \{1,z\}$ are disjoint, a contradiction. Thus, $[3]\subset F$ for all $F\in{\mathcal{F}}^*$ and the lemma follows.

We prove Theorem 1.8 for $r\geq k+1$ and $k\geq 4$ by the following proposition.

Proposition 6.2. $f^*(n,k,r) = \binom{n-3}{k-3}$ for $r\geq k+1$ and $n\geq \max \{4(k-2),k+r-1\}$ .

Proof. Let ${\mathcal{F}}\subset \binom{[n]}{k}$ be a saturated intersecting family. Let ${\mathcal{F}}^*$ be the family of $r$ -complete sets in $\mathcal{F}$ and let ${\mathcal{G}}=\partial{{\mathcal{F}}^*}$ . We claim that each member in $\mathcal{G}$ is a transversal of $\mathcal{F}$ . Otherwise, let $G\in{\mathcal{G}}$ be a $(k-1)$ -set that is not a transversal. Then there exists $F\subset{\mathcal{F}}$ such that $F\cap G=\emptyset$ . Since $G\in \partial{\mathcal{F}}^*$ and $r\gt |F|$ , there exists $x$ such that $G\cup \{x\}\in{\mathcal{F}}$ and $F\cap (G\cup \{x\})=\emptyset$ , a contradiction. Thus ${\mathcal{G}}\subset{\mathcal{T}}({\mathcal{F}})$ .

Since $\mathcal{F}$ is saturated, all $k$ -element supersets of any $G\in{\mathcal{T}}({\mathcal{F}})$ are members of $\mathcal{F}$ . By Lemma 5.1 (i) we see that $\mathcal{G}$ is intersecting. In view of Proposition 5.3, ${\mathcal{F}}^*$ is 3-intersecting. Since $n\geq 4(k-2)$ , by (1) we have $|{\mathcal{F}}^*|\leq \binom{n-3}{k-3}$ .

By using the Bollobás Set-pairs Theorem and the Hilton-Milner-Frankl Theorem, we prove Theorem 1.8 for $r=k$ and $k\geq 5$ .

Proposition 6.3. $f^*(n,k,k) = \binom{n-3}{k-3}$ for $r= k\geq 5$ and $n\geq k^3\binom{2k-1}{k}$ .

Proof. Let ${\mathcal{F}}\subset \binom{[n]}{k}$ be a saturated intersecting family. Let ${\mathcal{F}}^*$ be the family of $k$ -complete sets in $\mathcal{F}$ and let ${\mathcal{G}}=\partial{{\mathcal{F}}^*}$ . Define

\begin{equation*} {\mathcal {G}}'=\{G\in {\mathcal {G}}\colon G\notin {\mathcal {T}}({\mathcal {F}})\} \mbox { and } {\mathcal {E}}=\left \{E\in {\mathcal {F}}^*\colon \binom {E}{k-1}\cap {\mathcal {G}}'\neq \emptyset \right \}. \end{equation*}

Claim 9. To every $G\in{\mathcal{G}}'$ there is a unique $k$ -element set $H(G)\in{\mathcal{F}}$ which is disjoint to $G$ .

Proof. Let $G\cup \{x_i\}\in{\mathcal{F}}$ , $i=1,\ldots,k$ , the existence of $x_i$ is guaranteed by $k$ -completeness. Since $G\notin{\mathcal{T}}({\mathcal{F}})$ , there is $F\in{\mathcal{F}}$ satisfying $G\cap F=\emptyset$ . As $\mathcal{F}$ is intersecting, $F\cap (G\cup \{x_i\})=\{x_i\}$ for $1\leq i\leq k$ . Using $|F|=k$ , $F=\{x_1,\ldots,x_k\}=:H(G)$ is the unique possibility.

From Claim 9 it is clear that $H(G)\neq H(G')$ imply $G\cap H(G')\neq \emptyset$ . Define

\begin{equation*} {\mathcal {H}}=\{H(G)\colon G\in {\mathcal {G}}'\}. \end{equation*}

Let ${\mathcal{H}}=\{H_1,\ldots,H_m\}$ . To each $H_i\in{\mathcal{H}}$ , fix $G_i\in{\mathcal{G}}'$ satisfying $H(G_i)=H_i$ . Now $H_i\cap G_j=\emptyset$ iff $i=j$ , By (9), we obtain

(24) \begin{align} |{\mathcal{H}}|=m\leq \binom{2k-1}{k}. \end{align}

For each $H\in{\mathcal{H}}$ , let

\begin{equation*} {\mathcal {G}}'(H)=\{G\in {\mathcal {G}}'\colon H(G)=H\}. \end{equation*}

Claim 10. For each $H\in{\mathcal{H}}$ , ${\mathcal{G}}'(H)$ is 2-intersecting.

Proof. Suppose that there exist $G_1,G_2\in{\mathcal{G}}'(H)$ with $G_1\cap G_2=\{x\}$ . Let $H=\{x_1,\ldots,x_k\}$ . Since $G_1\in \partial{\mathcal{F}}^*$ , there is $F_1=G_1\cup \{x_i\}$ such that $F_1\in{\mathcal{F}}^*$ . By symmetry we assume that $i=1$ . By $k$ -completeness, we have $(F_1\setminus \{x\})\cup \{y_p\}\in{\mathcal{F}}$ for $p=1,\ldots,k$ . Since $|\{y_1,\ldots,y_k\}|\gt |G_2|$ , there exist $y_{p_0}\notin G_2$ . Since $k\geq 3$ , we may assume that $x_2\neq y_{p_0}$ . Then $G_1\cup \{x_1, y_{p_0}\}\setminus \{x\}$ , $G_2\cup \{x_2\}$ are disjoint, a contradiction.

Now we distinguish two cases.

Case 1. There exists $H\in{\mathcal{H}}$ such that $|{\mathcal{G}}'(H)|\gt k(k-1)\binom{n-4}{k-4}$ .

Since ${\mathcal{G}}'(H)$ is a 2-intersecting $(k-1)$ -graph, by (11) ${\mathcal{G}}'(H)$ is a 2-star. So let ${\mathcal{G}}'(H)$ be a star with centre $\{1,2\}$ . Let $H=\{x_1,\ldots,x_k\}$ . Define

\begin{equation*} {\mathcal {G}}_i^H = \left \{G\in {\mathcal {G}}'(H)\colon \ G\cup \{x_i\}\in {\mathcal {F}}^*\right \}. \end{equation*}

Note that ${\mathcal{G}}'(H)={\mathcal{G}}_1^H \cup \ldots \cup{\mathcal{G}}_k^H$ . Without loss of generality, we may assume that $|{\mathcal{G}}_1^H| \geq \ldots \geq |{\mathcal{G}}_k^H|$ .

Claim 11. ${\mathcal{G}}_2^H = \cdots ={\mathcal{G}}_k^H=\emptyset$ .

Proof. Suppose for contradiction that ${\mathcal{G}}_2^H \neq \emptyset$ and let $ R_2\in{\mathcal{G}}_2^H([2])$ . Since

\begin{equation*} |{\mathcal {G}}_1^H([2])|\geq \frac {1}{k} |{\mathcal {G}}'(H)|\geq (k-1)\binom {n-4}{k-4}, \end{equation*}

we have

\begin{align*} \left |{\mathcal{G}}_1^H([2])\cap \binom{[n]\setminus R_2}{k-3}\right |&\geq |{\mathcal{G}}_1^H([2])|-|R_2|\binom{n-k-3}{k-4}\\[5pt] &\geq (k-1)\binom{n-4}{k-4} -(k-3)\binom{n-4}{k-4}\\[5pt] &=2\binom{n-4}{k-4}. \end{align*}

By (10) we have $\nu ({\mathcal{G}}_1^H([2])\cap \binom{[n]\setminus R_2}{k-3})\geq 2$ . It follows that there are $R_0, R_1\in{\mathcal{G}}_1^H([2])$ such that $R_0, R_1,R_2$ are pairwise disjoint sets. Set $G_i=R_i\cup [2]$ for $i=0,1,2$ . Since $G_1\cup \{x_1\},\ G_2\cup \{x_2\}\in{\mathcal{F}}^*$ , we know that $E_1=R_1\cup \{1,x_1\}$ , $E_2=R_2\cup \{2,x_2\}$ are both covered by $k$ members of $\mathcal{F}$ . To avoid disjointness, we have $E_1\cup \{y_2\}\in{\mathcal{F}}$ for each $y_2\in E_2$ and $E_2\cup \{y_1\}\in{\mathcal{F}}$ for each $y_1\in E_1$ . Moreover, there is an extra element $z$ such that $E_1\cup \{z\},\ E_2\cup \{z\} \in{\mathcal{F}}$ .

Let $E_0=R_0\cup \{1,x_1\}$ and clearly $E_0\cap E_2=\emptyset$ . Since $E_0\subset G_0\cup \{x_1\}\in{\mathcal{F}}^*$ , $E_0$ is covered by $k$ members of $\mathcal{F}$ , we can find $w\notin E_2$ such that $E_0\cup \{w\}\in{\mathcal{F}}$ . For $k\geq 5$ we may choose $u\in R_1$ , $u\neq w$ . Then $E_2\cup \{u\}\in{\mathcal{F}}$ and $E_0\cup \{w\},\ E_2\cup \{u\}$ are disjoint, a contradiction.

Claim 11 implies ${\mathcal{G}}'(H) ={\mathcal{G}}_1^H$ and $G\cup \{x_1\}\in{\mathcal{F}}^*$ for all $G\in{\mathcal{G}}'(H)$ . Then by Lemma 6.1 we may assume that $\nu ({\mathcal{G}}_1^H([2]))\leq k$ . By (10),

\begin{equation*} |{\mathcal {G}}'(H)|\leq k\binom {n-4}{k-4}, \end{equation*}

contradicting our assumption.

Case 2. For each $H\in{\mathcal{H}}$ , $|{\mathcal{G}}'(H)|\leq k(k-1)\binom{n-4}{k-4}$ .

By the definition of $\mathcal{E}$ , we infer that each $E$ in $\mathcal{E}$ contains a $(k-1)$ -set $G\in{\mathcal{G}}'$ , implying $|{\mathcal{E}}|\leq |{\mathcal{G}}'|$ . By Claim 9 and (24),

\begin{equation*} |{\mathcal {E}}|\leq |{\mathcal {G}}'| \leq \sum _{H\in {\mathcal {H}}} |{\mathcal {G}}'(H)| \leq \binom {2k-1}{k}k(k-1)\binom {n-4}{k-4}. \end{equation*}

Define ${\mathcal{F}}_1={\mathcal{F}}^*\setminus{\mathcal{E}}$ . Note that each member of $\partial{\mathcal{F}}_1$ is a transversal of $\mathcal{F}$ . Since $\mathcal{F}$ is saturated, by Lemma 5.1 (i) the family $\partial{\mathcal{F}}_1$ is intersecting. By Proposition 5.3, ${\mathcal{F}}_1$ is 3-intersecting. If $|{\mathcal{F}}_1|\leq k\binom{n-4}{k-4}$ , then

\begin{equation*} |{\mathcal {F}}^*| =|{\mathcal {E}}|+|{\mathcal {F}}_1|\leq \left (\binom {2k-1}{k}k(k-1)+k\right ) \binom {n-4}{k-4}\leq \binom {n-3}{k-3}. \end{equation*}

Otherwise, by (11) we have $[3]\subset F$ for all $F\in{\mathcal{F}}_1$ . Then by Lemma 6.1, we may assume $\nu ({\mathcal{F}}_1([3]))\leq k$ and (10) implies

\begin{equation*} |{\mathcal {F}}_1|\leq k\binom {n-4}{k-4}, \end{equation*}

which contradicts the assumption and the proposition is proven.

Let $g(\nu,\Delta )$ be the maximum number of edges in a graph $\mathcal{G}$ with $\nu ({\mathcal{G}})\leq \nu$ and the maximum degree at most $\Delta$ . To determine $f^*(n,4,4)$ , we need the following result due to Chvátal and Hanson [Reference Chvátal and Hanson2].

Lemma 6.4 ([Reference Chvátal and Hanson2]). For every $\nu \geq 1$ and $\Delta \geq 1$ ,

(25) \begin{align} g(\nu,\Delta ) =\nu \Delta +\left \lfloor \frac{\Delta }{2}\right \rfloor \left \lfloor \frac{\nu }{\lceil \Delta/2\rceil }\right \rfloor \leq \nu \Delta +\nu . \end{align}

Proposition 6.5. $f^*(n,4,4)=n-3$ for $n\geq 63$ .

Proof. Let ${\mathcal{F}}\subset \binom{[n]}{4}$ be a saturated intersecting family. Let ${\mathcal{F}}^*$ be the family of $4$ -complete sets in $\mathcal{F}$ .

Claim 12. If there are $F_1,F_2\in{\mathcal{F}}^*$ with $|F_1\cap F_2|=1$ , then $|{\mathcal{F}}^*|\leq 35$ .

Proof. Without loss of generality, assume that $F_1=(x_1,x_2,x_3,z)$ and $F_2=(y_1,y_2,y_3,z)$ . Using that $(x_1,x_2,x_3)$ is 4-fold covered in $\mathcal{F}$ which is intersecting, the only possible extra 4-sets are $(x_1,x_2,x_3,y_i)\in{\mathcal{F}}$ , $1\leq i\leq 3$ . Similarly for $(y_1,y_2,y_3)$ we infer $(y_1,y_2,y_3,x_j)\in{\mathcal{F}}$ , $1\leq j\leq 3$ . Now consider $(x_1,x_2,z)$ , it is disjoint to $(y_1,y_2,y_3,x_3)$ . Hence its covering sets are $(x_1,x_2,x_3,z)$ and $(x_1,x_2,z,y_i)\in{\mathcal{F}}$ , $1\leq i\leq 3$ . Arguing in the same way with $(x_1,x_3,z),(x_2,x_3,z),(y_1,y_2,z)$ , etc, we infer that all sets $R\in \binom{F_1\cup F_2}{4}$ with $z\in R$ are in $\mathcal{F}$ . If ${\mathcal{F}}\subset \binom{F_1\cup F_2}{4}$ then $|{\mathcal{F}}^*|\leq |{\mathcal{F}}|\leq \binom{7}{4}=35$ , we are done. Otherwise we infer $z\in F$ for all $F\in{\mathcal{F}}\setminus \binom{F_1\cup F_2}{4}$ . Using $F\cap (x_1,x_2,x_3,y_i)\neq \emptyset$ and $F\cap (y_1,y_2,y_3,x_j)\neq \emptyset$ , we infer that such $F$ are of form $(x_j,y_i,z,w_t)$ with $w_t$ in the outside of $F_1\cup F_2$ . Now the intersecting property implies that $(x_j,y_i,w_t)$ cannot be 4-fold covered by $\mathcal{F}$ . Thus $(x_j,y_i,z,w_t)\notin{\mathcal{F}}^*$ . Hence $|{\mathcal{F}}^*|\leq \binom{7}{4}=35$ .

By Claim 12 and $n\geq 38$ , we may assume that ${\mathcal{F}}^*$ is 2-intersecting.

Claim 13. ${\mathcal{F}}^*$ contains no sunflower with 3 petals and a centre of size 2.

Proof. Assume that $(1,2,3,4), (1,2,5,6),(1,2,7,8)$ form such a sunflower in ${\mathcal{F}}^*$ . Since $(2,3,4),(1,5,6)$ are 4-fold covered in $\mathcal{F}$ , we may assume that

\begin{equation*} (2,3,4,a),(2,3,4,b),(2,3,4,c),(1,5,6,p),(1,5,6,q),(1,5,6,r)\in {\mathcal {F}}. \end{equation*}

The intersecting property of $\mathcal{F}$ implies that (by symmetry) $(a,b)=(5,6)$ , $(p,q)=(3,4)$ , $c=r$ . Since $(1,7,8)$ is 4-fold covered in $\mathcal{F}$ , let $(1,7,8,u),(1,7,8,v),(1,7,8,w)\in{\mathcal{F}}$ . Then one of $u,v,w$ is not in $\{5,6\}$ , by symmetry assume $w\notin \{5,6\}$ , it implies that $(2,3,4,a),(1,7,8,w)$ are disjoint, a contradiction.

Let $F\in{\mathcal{F}}^*$ . For any $P\in \binom{F}{2}$ , define

\begin{equation*} {\mathcal {G}}(P) =\left \{F'\setminus P\colon P\subset F'\in {\mathcal {F}}^*\right \}. \end{equation*}

By Claim 13, $\nu ({\mathcal{G}}(P))\leq 2$ . By Lemma 6.1, we may assume that ${\mathcal{F}}^*$ contains no sunflower with $5$ petals and a centre of size $3$ . It follows that the maximum degree of ${\mathcal{G}}(P)$ is at most 4. Then (25) implies $|{\mathcal{G}}(P)|\leq 2*4+2=10$ . Since ${\mathcal{F}}^*$ is 2-intersecting, we conclude that for $n\geq 63$ ,

\begin{equation*} |{\mathcal {F}}^*| \leq \sum _{P\in \binom {F}{2}} |{\mathcal {G}}(P)| \leq \binom {4}{2} *10 =60\leq n-3 \end{equation*}

and the proposition is proven.

Theorem 1.8 follows from Theorem 3.1 and Propositions 5.4, 5.5, 6.2, 6.3, 6.5.

7. Concluding remarks

In this paper we considered intersecting $k$ -graphs ${\mathcal{F}}\subset \binom{[n]}{k}$ . For a positive parameter $r$ we called an edge $F\in{\mathcal{F}}$ $r$ -complete if all $G\in \binom{F}{k-1}$ were contained in at least $r$ members of $\mathcal{F}$ , including $F$ . For $r=1$ this condition is automatically satisfied. For $r\geq 2$ we defined $f(n,k,r)$ as the maximum of $|{\mathcal{F}}|$ for families with all edges being $r$ -complete and $f^*(n,k,r)$ as the maximum number of $r$ -complete edges in $\mathcal{F}$ . The inequality $f(n,k,r)\leq f^*(n,k,r)$ is obvious. For $2\leq r\leq k$ all edges of the family

\begin{equation*} {\mathcal {L}}(n,k,r) =\left \{F\in \binom {[n]}{k}\colon |F\cap [2r-1]|\geq r\right \} \end{equation*}

are $r$ -complete. We showed that for $n\geq n_0(k,r)$ , $f(n,k,r)=|{\mathcal{L}}(n,k,r)|$ and for $r=2$ or 3 even $f^*(n,k,r)$ shares this value. However, for $r\geq 4$ , $f^*(n,k,r)$ is much larger:

\begin{equation*} f^*(n,k,r) = \left (1+o(1)\right )\binom {n-3}{k-3}. \end{equation*}

In the case $r=2$ we exploited some connections with the Erdős Matching Conjecture and succeeded in proving the statements with a linear constraint, $n\geq 28k$ . However for $r\geq 3$ our proof requires $n_0(k,r)\gt k^{2(r+1)k}$ .

Problem 7.1. Does $f(n,k,r)=|{\mathcal{L}}(n,k,r)|$ hold for $3\leq r\leq k$ and $n\gt ck$ with an absolute constant $c$ ?

Another open problem is to determine the exact value of $f^*(n,k,r)$ for $4\leq r\leq k-1$ and $n\gt n_0(k,r)$ .

As the analogous problems for $t$ -intersecting families, we can define two more functions.

\begin{align*} &f(n,k,t,r)=\max \left \{|{\mathcal{F}}|\colon{\mathcal{F}}\subset \binom{[n]}{k} \text{ is}\ t\text{-intersecting and } r\text{-complete}\right \},\\ &f^*(n,k,t,r)=\max \left \{|{\mathcal{F}}^*|\colon \begin{array}{l@{\quad}l} \exists{\mathcal{F}}\subset \binom{[n]}{k} \text{ is } t\text{-intersecting},\ {\mathcal{F}}^*\subset{\mathcal{F}}, \\[3pt]{\mathcal{F}}^* \text{ is relatively }r\text{-complete in }{\mathcal{F}} \end{array}\right \}. \end{align*}

Example 7.2. For $n\geq k\geq t\geq 1$ and $1\leq r\leq k-t+1$ define

\begin{equation*} {\mathcal {A}}(n,k,t,r)=\left \{A\in \binom {[n]}{k}\colon \left |F\cap [t+2r-2]\right |\geq t+r-1\right \}. \end{equation*}

By essentially the same proof as in Sections 3 and 4, one can obtain the following two results:

Theorem 7.3. For $k\geq 3$ , $r\geq 2$ and $n\geq n_0(k,r)$ ,

(26) \begin{align} f(n,k,t,r) = \left \{\begin{array}{l@{\quad}l} |{\mathcal{A}}(n,k,t,r)|,& 2\leq r\leq k-t+1;\\[5pt] 0. &r\geq k-t+2. \end{array}\right . \end{align}

Theorem 7.4. For $k\geq 3$ , $r\geq 2$ and $n\geq n_0(k,r)$ ,

(27) \begin{align} f^*(n,k,t,r) = \left \{\begin{array}{l@{\quad}l} |{\mathcal{A}}(n,k,t,r)|,&r=2,3;\\[5pt] \binom{n-t-2}{k-t-2}+O(n^{k-t-r+1}), &4\leq r\leq k-t+1;\\[5pt] \binom{n-t-2}{k-t-2},& r\geq k-t+2. \end{array}\right . \end{align}

Acknowledgement

We would like to thank the referees for their helpful comments and corrections.

References

Bollobás, B. (1965) On generalized graph. Acta Math. Acad. Sci. Hungar. 16(3-4) 447452.CrossRefGoogle Scholar
Chvátal, V. and Hanson, D. (1976) Degrees and matchings. J. Combin. Theory Ser. B 20 128138.CrossRefGoogle Scholar
Erdős, P., Ko, C. and Rado, R. (1961) Intersection theorems for systems of finite sets. Quart. J. Math. Oxford Ser. 12(1) 313320.CrossRefGoogle Scholar
Erdős, P. and Rado, R. (1960) Intersection theorems for systems of sets. J. Lond. Math. Soc. 35(1) 8590.Google Scholar
Frankl, P. (1978) The Erdős-Ko-Rado theorem is true for $n = ckt$ . Coll. Math. Soc. J. Bolyai 18 365375.Google Scholar
Frankl, P. (1978) On intersecting families of finite sets. J. Combin. Theory Ser. A 24(2) 146161.CrossRefGoogle Scholar
Frankl, P. (1987) The shifting technique in extremal set theory. Surv. Combin. 123 81110.Google Scholar
Frankl, P. (1991) Shadows and shifting. Graph Combin. 7(1) 2329.CrossRefGoogle Scholar
Frankl, P. (2013) Improved bounds for Erdős’ matching conjecture. J. Combin. Theory Ser. A 120(5) 10681072.CrossRefGoogle Scholar
Frankl, P., Kupavskii, A. and Kiselev, S. (2022) On the maximum number of distinct intersections in an intersecting family. Discrete Math. 345(4) 112757.CrossRefGoogle Scholar
Frankl, P. and Wang, J. (2022) On the sum of sizes of overlapping families. Discrete Math. 345(11) 113027.CrossRefGoogle Scholar
Frankl, P. and Wang, J. (2023) Intersections and distinct intersections in cross-intersecting families. Eur. J. Combin. 110 103665.Google Scholar
Gerbner, D. and Patkós, B. (2018) Extremal Finite Set Theory. CRC Press, 1st edition.Google Scholar
Hilton, A. J. W. and Milner, E. C. (1967) Some intersection theorems for systems of finite sets. Q. J. Math. 18(1) 369384.Google Scholar
Katona, G. O. H. (1964) Intersection theorems for systems of finite sets. Acta Math. Acad. Sci. Hungar 15(3-4) 329337.Google Scholar
Katona, G. O. H. (1974) Solution of a problem of Ehrenfeucht and Mycielski. J. Combin. Theory Ser. A 17(2) 265266.Google Scholar
Kostochka, A., Mubayi, D. and Verstraëte, J. (2017) Turán problems and shadows II: trees. J. Combin. Theory Ser. B 122 457478.CrossRefGoogle Scholar
Kostochka, A., Mubayi, D. and Verstraëte, J. (2015) Turán problems and shadows I: paths and cycles. J. Combin. Theory Ser. A 129 5779.Google Scholar
Kostochka, A., Mubayi, D. and Verstraëte, J. (2015) Turán problems and shadows III: expansions of graphs. SIAM J. Discrete Math. 29(2) 868876.CrossRefGoogle Scholar
Liu, E. L. L. and Wang, J. (2020) The Maximum number of cliques in hypergraphs without large matchings. Electron. J. Combin. 27(4) P4.14.CrossRefGoogle Scholar
Wilson, R. M. (1984) The exact bound in the Erdős-Ko-Rado theorem. Combinatorica 4(2-3) 247257.CrossRefGoogle Scholar